You are on page 1of 185

.

!
$%" &' + $%# &( + $% &) + "
* $+

, -.-( -.-( /, 0
-.-( /,
1 23 = " # $ . = " # $ .

/45 46 !
# 7.
87.

!
7 #9
# :8. !
7 ;$
!
7 ;#

4 3 3
* 0 )
_____________Download from JbigDeaL____________

$)

' $.

(
)
# 2

"

$
)

2 %
-
#

!
!7 7. $
7$ $
;$ < ;#
*
(
* !( + )& − = && 3
0
*

* 3 >? $ ≤ >?≤ ".. < >< ?


"

*
?
_____________Download from

@ 0

4 AAA2-- /
B AAA('% C
/,+
AAA%'% D
E D5#"#3 5 F ? G

-.-% !

4 DC=
I* / ,

B 6K L C , =
K
MK6E D ,
E DA4EN 5 , 0

4 B / D 3
B C 5
/ D @
E H 5 I

4 * J
/
#
B * # 0 "
.+ $+ C

* > #
D

E * % 5
# .+ $ #
+$
_____________Download from

"
!
* @K5 @K5 !

4 0 !

*
@4D

* *D4/ -.-) !
O*4

#.M
OK/

* 46A -.-) !
=
K
3
- 4E(34E.
*D4/
O
L ? +
I !
( ) #

( )
( + )
( )
( ) #

* 0 ?!
/,5M $. /,5M #. /K/ /,5M $. /,5M #. /K/ /K/ /K/
/,5M #. /K/
* 0 !
#. $. #. $. #.
#. #. $. $. #.
$. #. #. $. #.
#. #. $. #. $.

/ !
P @! ! 9
! 9

7 .9
&:@ 9 7<$
P97

* P O O

( )
( ) +$

+$

4F ? G !
3
_____________Download from

?
?

!
D
*

O / /

!
*

4 0
3 3 3
* 6D,

* 0

$ # "

$ ∧ # → " $ ∧ $ →Q #

!
B $ #

* > $ ∧ #

O
O
O

6 = $ ($ + . ) & = $$ &. = $ &. 1


R
( )⊆ ( ) ( )⊆ ( )
( )⊆ ( ) ( )⊆ ( )
_____________Download from

( )⊆ ( ) ( )⊆ ( )
( )⊆ ( ) ( )⊆ ( )
O

1
R

* 3
3

2 I 0 !
/ !
8) !
7
!
7 9
/ B9 0 /

K 0
( $ # ") = ( )
4
4 RA

) 4 !

. .
. $
$ .
$ $
C
_____________Download from

( )= (. # - $. $)) + (" $$ $# $2)


1 3 +
? 0
4 3 4OE
$. 4 SL 3

S. C. S$ C$ S# C#

L. L
L$ L L# L

% , E 3 3 = 3
!
? 6L
* 4B
5 5
=5M *
E $.#2 3 = 3 $L ×
$ D4@
DA4E=1D *A N 55 @5

( 0

4 -
I 0

- -.-) !
0 !% @M
DK@ !.... .(
_____________Download from

D4@ !$... $( !
DK@ 0
D4@ 0
E !
KD 4 ##
E D@
4 /,
!
4 7 )) B 7 44 7 ( E 7 $. M 7 $. 67 / 7 .#.. 5/7 $( K
E ? -.-)

? *
=
K 2.M
O*ADD,/* B, ,66
* =
K
2.M 4
55 @5
1 #.. M
=K "2.. M

/KD*3E4*AK,*/,*

K /KD* 2. M

O*ADD,/*

B, ,66

5*DKBA O/,* DK@ EAH A

' 3
!
/ / 9
!
7#9
!
7 < 9
T/U

!
7$
!
7)9
_____________Download from

!
7$..9
/ & 9
1 V7+ 7+
!
E /

H ! 9
/ /9
!
7)9 !
7$.
T/U9
C9
! 9
/9
TCU9

!
7$9 !
7#9
C9
1 V 7+ V7+

$.
; = !
→ −! !
!
! →

→ ( )
A
R
; = =
3
E
= 3 = R
E
_____________Download from

$$
$" $#

$9

WX
!7 %
W
>X W
?X

>! W
>X

WX
!7 %
≠ # %
WX!
9
?! W
?X

WX
!7
≠ # %
WX
!7

7 !
7>9
WX
!7
3
9

E R B

$# 5 !
5,//6AD 5 KEA 5O4@A *N
/4D* / KEA /O4@A /EA5 *N
/DKSA *5 /D KEA /DO4@A *N
5/DD 5 KEA / KEA /D KEA CSN
_____________Download from

1 5C6 0 !
/ / KEA > EA6M
EA6M
/ *N / KEA *N
>
*N
1 !
I 5 KEA > /D$ /D#
I /D KEA >

$" I 3 0
? ? ? / I
3

$2 !
$

$-
)
#.
#)
( $"

#(
$$ ' $) $(

1 R
I ? )

K 3
1 3
3 R

$) 5
&
!
( &) ( ') ( & ') ∧ ( &) ( ') ( ( & ') →Q ( ' &) )
∧ ( &) ( ') ( () ( ( & ') ∧ ( ' () → ( & () )
∧ ( &) Q ( & &)
_____________Download from

43 0 A3 0
E R
R

$%
# 0 $+
.+ .+
$+
5

$( 5 *
?
6 6
$ I 3 6M
0
RS R
!!!!!!!!!!!
"
9

9 9

" #
$ % & ' !!!!!!!! #

!!!!!!!!
( & ' &
% ' & !!!!!!
( &

!!!!!!!!!
) &
* +, &
& !!!!!!!!!!
, - . $
% /0 ' 1
#2 .
!!!!!!!!!
( %
!!!!!!!!!
!!!!!!!
% " %& +

!!!!!!!!!!
% ( % !!!!!!!!

2 " &
&

)34 /5 67 /5 3)34
) . &
6% /5 ( %
8 /5
_____________Download from

" &

+ & ' * +

& * +"0:

0"8; <9<1
8; 0" & &
* +

== = = ==
== ==

> * +

6 6
>& .
>

? 30 @
8
9A

)2 #
2 #

2 # B
$ 1 C D

)
B -C C -$
9 -1 B -1

#
%
9
8 E ( 5 %
_____________Download from JbigDeaL____________

% ) D
29 0 1 F

$2 2 D 2

% ) & C -
6 & # %
&
2 # $

% ? % @
%
& & & &
) G
3

% % - /;;/? )
& ) H I
? ; /;;/? ) 5 /;;/? ) &
H I &H I H I &
/;;/? )
? A @
/;;/? ) /;;/? ) &
/;;/? ) /;;/? ) &
)
)

% " >;5 ;);5 %


? A @
&

&
&

% ? A
( ∀ ) ( ) ∨ ( ∀ ) ( ) → ( ∀ ) { ( ) ∨ ( )}
( ∃ ) ( ) ∧ ( ∃ ) ( ) → ( ∃ ) { ( ) ∧ ( )}
( ∀ ) { ( ) ∨ ( )} → ( ∀ ) ( ) ∨ ( ∀ ) ( )
( ∃ ) { ( ) ∨ ( )} →J ( ∀ ) ( ) ∨ ( ∃ ) ( )
_____________Download from

% ? A &
∨ → ∧ ∧ → ∨
∨ → ( → ) → → ( → )

% ? % @
(K) = K ( K K) = ( + )K
( + ) K= K+ K K K= K+ K

% ,L % ;L -
L- L " & @
2 2 M 2

%% " %

%% , @
H & % '
2 . H

04)

# ?
?
@
()"5,( -7 ()"5/ (2- 7
, 6E-7 , 6E- 7
( 7M (27
634" 634"
, 36- 7 , 36- 7
?/54 7 ?/54 7 M
634" 634"
634( 634(
L %

(
_____________Download from

&

8 &
@

)
E , N= O -
E %
? &
. & @

$ " + ) &
& 6%
+
) & )- "
( - 9) &
) ) & & "
&, 99
& $ %
4 # 4
&

1 & " &-


& - 199 19 - &, <9P
& '
& 9B &
& 4
&
)- " -
A
' ,35- ,
3 52 ,3 5# &
,
35 Q & ,
3 52 Q & ,3 5#
4 & - -
0 & ) Q0 & Q0 & "

C ) & & 29
& ' 21C & 5)(- 21C
& 605/(- C ,A
/ & ,A
/ & 665)(
& 5)(
_____________Download from JbigDeaL____________

4 &

& >. >


5)( C ×< $9
605/( 21C ×< 19
665)( 21C × < 199 #R
21 8. '
& & )
<9<1- % ' 665)(

D "

S #
S

" 9
GS T
" "M
S K"

N . O
4
>

< ; 4 L ) &0 % &


L 0N
FO % F- 0

L & - L
- ∈ −
&& &
4 0)>"); T )
&

B > >
>
& >
_____________Download from

(6( 65 7 " 7 >


,
34 /36 > ,> &- > &
&
)44 7 ) % >
46;6 6 7 4 % >
L % & L
6 U &
& . " & & .

9 ? & & - & -


& - & '
0 )NO 0
) &@
" - &.
2 ) . )NO
# 6% & 0 . )NO
$ > )NO .
, @6% &U
&
> & G
% & %
)&

0 V
0 )V
0 V
0 )V
==
V

& )V
V

V
)V

% )V
_____________Download from

? & %
& & &
& 5
&
4 ;V

; ;-
;
? # '
% ' & 0
% )

M Q M Q W

2 4
>

> 2 >$
V

>#V

>1V

>C > <


6

V
>D
0 V
>B

;
$- & S9 X- -
Y- S 9- S -
S 2
&
_____________Download from JbigDeaL____________
www.estudentzone.com

# 8 & @
8
3
5
?
>
56E:63 > ":> /(65- 8/ 6;
>65F6> 8/ 6;- >3)" >
;, 6> ":> /(65- >3)" >
-
&
6% ' &
5

$ ,L - % ≠ -
& - 2
=
" { -2-#- $- C- <- 2- 2$} &
;"( L"4
? @

1 : ( ( - → = )

( )→ ( ) J ( )
2 ( )J ( )
# ( )
$ J ( )
? % & &' - 0- E- 5

; > Q % 4 -
∈ - 5 > 5 '
' S1

C ? @0 &

{ ′′ }
5
_____________Download from

" #
$ $ α
$ $ $ α ≠$ ( )
$ $ $

($ $ α) (α $ $) ($ $ ) ($ α $ )

% "
%

%
+ + =$ &

'

( ) * # ( ) !

% ( +

+ ,-."./0 1
2- ( 3 / 32 444
.5/2 3 6 $ 7- "- ( 3 $

6 , %π !
$ − < <
( )=
$< <

+ ( π − ) − π
+ π %

8 # 3π

+ + + +
(% 6% %

π% π% π% π%
+ 9 : %

9 1
_____________Download from

∞ ∞ −
$ − $ + $ + % $ 6

%
" ( )= %
−( +% +

#
#
#
#

% % % $ ;;;;; !
! 2 !
/ #

%
( − ) + %( − )
;;;;;;;
→$
( − )

%% "

(( ) < (
& ;;;;;;;;;;;;;;
( <)
(

%( ! !
= %
+ − %

" ! ω ' ;;;;;;;;

%+ 7 ′= − ($) = $ "
($ ) ! !
. '= > 3$ ;;;;;;;;

%6 , ? 3 +$ ,-."./0
6 $4 (
= −% 4 4% + + ;;;;;
4( +

%9 " ;;;;;;
$ $ + %
_____________Download from

$ $
$ − $ −
% = # +
! @ ! !'A
$ $
$ $ $ −
;;;;;;;;;

%: 7 = %
+ %
+ %
)
#

;;;;;;;

%B " + =$ C !

($) = $ (λ ) = $
' λ
;;;;;;;;;;

% $ " D ( ) !

( )= (% − )π ≤ ≤% π ( = % ( )
$

;;;;;;;;;;;

π %π (π +π 6π 9π π :π
$
_____________Download from

" "A.55 ) E 9 :
$ : 5 % " !
' @
!

9 ! !

#!

# -. # 0-. 0/02
0-. 0

9%
!
= + = + =$ =
+ =

9( , $$$ ! F
'=
' &

F E F E% F E

= = =

) . '( @ '9 @
'% @ 0
_____________Download from

9+ / : "
! %+$$

$B % 9 % : % B
%+$

96 @

$ $ )% = )$
:) $
= )

99 / .- " ! !
+' " > .- ×
" × 0
0

9 /

0 G 3 %$$
2 3 %+$$
" ! 3 9% 6$$
" ! H E
G
H E

9: " !

$J

#
#
$J :$J
$J

) . @HI
@HI
)
5#

7 !
@HI #
_____________Download from

, ' ( H/)@/D
H H/./ K
& K
H # !& K
& K

&3 K
# & 3! L
K
E #& K !& K

#&3!L K
K

&3$KH K K
&3$KE 4 K

" ! H/./ &


H
$

$ # ! C ! $
# ! C
H

% " ! H/./ &


$ # E
$ ! E
# E
! E

( 1
H/./ H E H/./ H H/./ E
H E
_____________Download from

+ 1
& K

&3 L K
&3 ' K
&3 '
K
#

6 , 1
K
! 3M K
3
& K
&
K
& K

K
&3 K
M &36 %K

"
"
" 6%
"
0

9 / ' &

! !

0
_____________Download from

/ !
! &
#

: " !
# ! #
!
# ! ! !

B @ ! ! "
! ( / 8 @ ( + 9
! , 1
B $ (
6

$ / C # !

F @HI 8

" C $+
C

+ $ %
0

: @ 7 ' N% "
1
7 !
" ! ! 7
7 !
" ! ! 7
0
:% " ∧ (O ∨ ) &
! ! ∧
! ∨

:( D ) ∪ % ∪ ∪ = "

:+ D / > "
× &

(% ) (% )
% %
%
%% %

:6 " ' P
! #
! #
! #
! #

:9 D / 8 ! " '
C / 8 P &

: ( ) ( ) &
≤ ≤

( ) ( ) %
( )(
(( ) %

( 6 %
)
_____________Download from

" "50 5 6
/

B / ! ! ! ) !
!8 !

$ " @HI &

-H @-25 -25 . .

- !

. . ! . . @HI
3% Q
. .
. . ! *
/ ! %$$$ 3% . .
H@
1
/ 'C H@
# 1

' ! !
!

A !D / " H ! H "
( ) ( ) ( ) ( )
$−9 $ BB$$$ $ $$

$−6 $ $$BB: $
%
$ $$$$% ''
( $−+

@
! !
_____________Download from

% " H '
! $ − $ − ,
# !
/ # C
!

#&3 &3$ C&3$K

/R CS
N # #&3/R CS
C&3CL

C&3$K
&3

( @ ! " %

D ? C' ≤C≤ C
/ T 7

% −

+ / )/ # > 9+ !
! > + ! / % !
> 6 % ! 9 = >
# A !
! #1

6 @ &
K
C& #K & K

&3 '
/RSN / RL S

& /RS
K
/RS
&3/RL S
K
_____________Download from

/RL S
&3
K
'

D QU U *) #
) !
− )

9 H ! ! !
! ' %
'

- % B $ & 6
A ! 1

: ) @
∧( → ( ∨ )) ∧ ( →O )
I

" "50 5 6
/ ! ,V5

B / ! 0/02 W !
/ 8 ! 2 ' H
2

8
8

2/ /

@
H

28 8

/ /
_____________Download from

%$ 7 :$:6 &
! !
D !
DAD2 6$$$
V 8 6
75"& 0 %$
-V /
0? A
2@. 8
F
0X 75"

% " !
&
% ( + ( 6 % 6 + ( % (
" ! !
! > +

D.I
,,-
/ ! ! !

%% '
9

)% )(

)+ )6

)9

!
"
/ ! !
!Q
4*
_____________Download from

) 4
4
4 4
4 4

%( )ED ! /
! .

%+ ,
&

( (0, 1
! ( 0, > !
( 0,

%6 / H !
#&3 #
&3 # #
K
UK
#' @,7
/ # ! 2
4 @,7
) &
C&3% $
#&3 # K
!&3 # %

%9 /
H &
H #& K!& K
H ! KH
) !

! H
# ! 1
_____________Download from

% 2
{ } (

%: D ({ { 4 ) 4 = ) &

4 = 4
4 =
_____________Download from JbigDeaL____________

! "
# $ $
# % $ &
"
# ' $ # & ( $

)) *+, , & & % & '


" # '
' " %" & # '
( # ' &
% ' & & &

)- . % ' "&& & / ×


$ 0

′ =) ' = −)
1 % ( )′ =

)2 ( % & % # $ % = ( 3 )
% '"3 $
+) = + ( 3 ) ' +) = + ( +) 3 +) )
+) = −) +- ( 3 ) % +) = () + ) ( +) 3 +) )

)4 . % ' " ' " # 3 3 $ 0


( ∩ )= ( ) ( ) ' ( ∪ )= ( )+ ( )
( )= ( ∩ ) ( ) % ( ∪ )≤ ( )+ ( )

)5 % 6
$ 7 & & '
& ( & % # " $ &
' $ & $ &
& & " % $ &
% & $ $ $ & %
_____________Download from

)8 &' % & $ %
)5 ' )9 : % ;

): & <" ' "

( )= + 3 ( ' ( )=- + 3 (
- -

( )= + $ % ( )= +
- -

)= $ < $ '
+ +
' + +
=
?
% + +

@
9 9 −2
); ( & < ; 2 5
2 ) )

9 ' ) - % 2

) )9 & $ ( 3 ) ( ' ' 3 $


0
−)
= # " ∈ ' = -
# " ∈

( )
-
= - -
# " 3 ∈ % %

) )) & $ %& " $


& & ' # % $ / / × 3 >/
& & $ %
3 $ & 3 % < (3 ) &
$ & <

+ ' + −) +
( − ))
% +
( − ))
- -

) )- & % % ' % ' & & %


& '
& ( & & < " % " < "
_____________Download from

' " <>% % ' & % % $


' " % &
% ' $ % & % " & $ #
$ $ $ &

) )2 & & " $ $ #" % # ' /% # " "


% & #%
.,A $ & $ & %
' **+ $ & $ & %
.*A $ & $ & %
% ' #

) )4 $ & ' ' % &


% $ ( & $ )3 -3 23 43 5
% 0
23 43 53 )3 - ' 23 43 53 -3 ) )3 53 -3 23 4 % 53 43 23 )3 -

) )5 &' ' $ $ # ' & % &


$ $

' - ( − ))
%
( + ))
- -

) )8 $ # ' $ & "0


,$ $ && < $ &&
' >% & * % & *
>% & B@ % & B@
% >% & $& % & $&

) ): . (% ' % $
' & 0
' "
,%< % B " & &

) )= $ ' % '" <> $ && 0


" < > > &
' " < # %
# ' ' % %'
% C ' & ' " $
) ); $ $ & % $ % (
$ &0
@" & $ && $ ' ( ( $
@ #% % % %"& %

) -9 $ ' ' %
>'" % >'"> & & $& % 0
B $ # &
' B $ %% " &
B $ " & &
% B $ " $ &

) -) $ & 0
D % & % &' "
$ $ $ &
' +# " $ % $ & $ %%
&
C & & " ' % && % $ & $
%% &
% ' # $ # $ $

) -- $ & 0
+ & ' " ' & % " $
%" & $ && $
' % > ' % % # % & $
# < % < ( # ' " 3 ' "
' " %
% @ > ' % % % & $

) -2 ? % $
2
9≤ ≤ )93 999
) ( )= -
≥ )93 999
9≤ ≤ )99
( )=
> )99
- 2

$ 0

) ( ) 9( - ( )) ' ) ( ) 9 ( )
2

- ( ) 9( ) ( )) % - ( ) 9( )
_____________Download from JbigDeaL____________

) -4 ? % $ $ ' ( $ %
% $

59 )59 299 259 899

$ %%

' ( / 2993 -53 )-53 59 ' %

' " "


' ' ' "
' ' "
% ' #

* ' (

-) &' > % ' "& %


EEEEEEEEEE

-- + > $ # $ 3 ' " EEEEEEEEEE


# % > && #

-2 & $ F <# "3 "&& "3 > "&& "3 # "G


, 1F <3" ∈ -
≠ G EEEEEEEEEE

-4 &' ' {)3-3 } %% % " EEEEEEEEEE

-5 &' %$ $ $ % $ % % # EEEEEEEEE
-8 ' ' " # ) ' ' " # %
$ - ' ' " % $ 2

' ' " # & )3 - % 2 EEEEEEEEEE


-: ? % >' % $ $ ' -H & & $
&' $ $ # 3 3 ' % EEEEEEEEEE
≤ ≤ EEEEEEEEEE
-= . 3 %?' % % # ' ' 9 =3 9 5 %
92 # " ' ' " #
EEEEEEEEEE
-; % $ & & EEEEEEEEEE
# % $ &
_____________Download from

- )9 $ < $ $ $ / % '" &


$ EEEEEEEEEE
9 )
9

) 9
?

2) * <
D < $ %% I
% =9=5 & %
< &

2- * <
< % $ % & && " & "% , ? ,% %
? & &

22 * <
*D * D ' % $ % %% $ "
' " $ ' " &' %$

24 D $ &

>, $
, $ >J ' , % $
> % +% "@
# & H, % " & .

25 D $ &

( > & ,$ <


.< " ' . ., $ &&
! ?? .. $ &&
#, # % $ % <> $ &&
28 *
" % & " >? %% & & ? *
%% % " # $

2: & , 3 3 ? % # '
? " % %% (" ,0

2= # $' < $ " & & & &'


& ( ∪3 − ) # ∩

2; # " ' ' '


' # &

2 )9 D $ &

?. A K # " $ % % ' K $ %
%
' A K K& % % % ( $% "
. K $ % ?K ( $% "
# % 3& % ( $% "3 < #
?D+

2 )) *
.$ % % % # " % % %
) 9 )
2 )- * % # & < −) ) )
9 ) 9

2 )2 . % ' A $ " ' % %


⇔ % & "

4 . L' % % L = +
?1 L # % ' (
L 1 EEEEEEE
?L 1 EEEEEEE
' # $' # 3 %?
+ =)
+ =9
_____________Download from JbigDeaL____________

5 2> " # " % < " % A


% # &' # 2> " # %
- ( − )) + 2

8 <3 & % '" $ &0


* <3 $ $
C
# $ K
' $
# 1)
M9
' $
& % - 1)
# 1# L<K
# 1# L <L<3 % #- K
%K
#
%K

: " $ N
9O3 N
)O3 PPPPPP N >)O
% & " " " '"J 3
)≤J≤ >) & $ & % $ & 3 $
$# ' ? & $ & '" $ ' ( &
# ' ' "% %
& 1 K
19K
EEEEE%
' $
& 1 NO
K
Q1 K
EEEEE%
' $
N
QO1EEEEEK
Q1 Q
RJ & % K
Q
S&
& 1Q
K
%K
N R>J & % O1EEEEEK
1EEEEEEEEEEEEK
%K
_____________Download from

= % )- % % &' % ) )- > %
# % > % 3 % # % % 23 53 43 -3
:3 =3 83 )93 ))3 )-3 ;3 )
, $ & & 3 3 %
% 3 % % $ && "

!"# $
"

; * $: ' $ $ && $ % % & $ #%


$ '

: 8 5 4 2 - ) % #
& & & &

@ & & $ & $ & ) ' % '


% & $ % ' %
$# & $

)9 $ & =9=5 &' " $ $ %% )8>' $ % ?@ =>


4>->) "? % % @ & &' & % ?
%@ ,$ %' % $ ? $
% $ $ $ % %$ %

)) * % > - 3 ) % 9 $ 3 $# $
( ( & & - ) 9 = 999 "

T U- @ U) U9

?J ?J ?J

J 9

)
? (

* ( $#

)- & ?BA " $ ) % - % "


$ # ' 3 K{ ← ⊕ 3 3 = )3 -}
_____________Download from

A $ V+, 3 % % < $
$ ) % -

' $ ( ) @ &
%

> >)

;4>:

)2 ? % $ &
?+A, 3 &
A@ 3 & 3 $3 "
, , @ *+, 3
% %' % & "( " W
" H
' A@ % " %
" %" $ * " 3 %"
$' "
B &' % # $ % 2--
' U. "
B $ %" " $ % "

+
)4 ? % − % % $ 0
+
− % % ' % % -% ( " %
+
@ $ − % $

-5 2= 8; ;2

)9 )5 -5 28 2; 4) 4: 8; :) :5 =2 ;5 ;; )9) )95

+
' * − % % % 3 &' % %
% $ 9 (−)
_____________Download from

)5 A $# #
−)
(- + )) = -

=9

! & % & " %

) 4 ;
' A ' % #

=
( + ))
=) -

)8 # " & & $ ( 3 +3 9 ) 1

@ % $ && #
): $ & * "

? Q & '
' &' & % '
&' "

)= $ & * "

' " ' % & &' "


$ $ $ &
' "&' % % W< H &' " $ $ $ % %%
% $ & '" &'

); #
={ <>' ( 5X % & < π}

<>' ( & <& ' ( < # 5H


$ & 0 %
' $#
$
# " & '
# " & '
# #

' # $ $ .) $ % $ $ ) ∪ - $ 3
$ $ - " $ 0B # "
_____________Download from

-9 $ && ? & ("> & * & ? * %


& ? 3 3 % ?3 > & % &
?* ? * % $ . $ 3 $
% # 0

-) ? % $ #
' ) $ K $ K
' $
19 1) ' 1)
' 1 ' >) R ' >-
%K
' # & ( 84 '" & $
" %% % & % % (3 % $
# % %% ( - '" 3 & & <& & #
( # # "

-- ? % $ &'
B $ && K
# $ K
% K
' $ %K
% K
# $ K
' $ 15 K %
' $ 1-K
K K K
%
% '" ' # $ &
$ & % # ' K
@" & $ & % # '
# "

-2 # & $ $ % "
$# '
.+ @ %% K % $
K & %%
+, %% K $
K %%
@@ %% K %% $
K %%
K %
K $
_____________Download from

? % $$ &&
% 1 + ( )
" <% % $ % $ $ &&
& % ' % ' #

-4 % % $ ' # #
' % '" %$ & & $ %"
$ & % & <& & % % $# '
C # K 1φK
C ≠ φ%
' $
# < K∈C
C 1C Y F GK

) 1 !∪ {"}

%K
K
? & $ & '" " $ " # <
' & & <" $ &

-5 " $ >% $ % 3 ) ≤ - ≤ ≤
@ ' 3 $B ( % % 3 & $ & % 3 Q
3
+ = $# $ D3 3Q<

-8 U $ & % & " ( $# %


& & (3 &
(3 % % & #%
' % + & $ " &
%% $ " ' & %
% ( @ 3 3 B % B @ & "
& & (

-: @ % $ $ % &
&' % & ) 8

) %
- 1)
2 M $ <
4 1 R)
5 1R)
8 <
_____________Download from

' ? $ ' # % $ & $ ' $ > %


"0

-= ? % $ $# $

-
)

2
-
) 9

* % " & % % (
' + 3 %
JbigDeaL____________

!
"
# $ %&' ()* # $ +' )(* # $ %&' )(* #,$ +' ()*

- "
# $ &(. & ,/
- !
# $ 0 1 ,
# $ 2 3')(
#,$

4 ' ,
# $ ,, , 1, !
# $

# $ ,, ,
#,$

5 ' 0 6 %
1 "
%7
89 : ;7
8 :
' %<; %= %7
8 #%$ %7
8 >%:
# $ # $ 9 # $ 9 #,$ +

.
# $ ? , , ! # $ @
# $ * #,$ A ,!

B !C
# $ # $ ?.
# $ 0 #,$ . !

D ' , , ! 6

# $
# $ ,
_____________Download from JbigDeaL____________

# $ , !

#,$ E,

- @ ,!E
!"
# $ ( # $ 3&F
# $ )')( #,$ @ # $ ,# $

G ' 6 , ,
! ! , ' 3 ,? ,
,, !6 1 , , "
( ) ( ) ( ) ( )
+ > > >
# $ ∪ # $ ∪ # $ #,$

, ,
→ ∝ ∝
→∝
→∝
∝→
7
# $ 1 # $ &
# $ 1 #,$ 3&# $

- 1 ,! , "
,,16!
3 ,!
. 1
2 !
,
# $ H # $ ',
# $ #,$ )

- , , 6 6 6 6
"
# $ # $ 4 # $ 5 #,$

4 - , ! ,
1 0 "
' '' ''' =<
_____________Download from JbigDeaL____________

# $ ' # $ '' # $ '''


#,$

5 C , ,
!- , ,, C , "
# $ ( C ,
# $ &
# $ + , 1 ! , , C ,
#,$ ,, !

- , ! , ! "
# $ 2 I ) # $ & ,&
# $ ) ) 2 #,$

B ) , J , , J K 6
,
# $ ( ) # $ ( )
# $ ( + ) #,$ ( + )

D ! , , , 7
# $ # $ − # $ #,$

! , , , GGG #
$ , D 7
4
B G D
# $ # $ # $ #,$
D

G 3 & ! ,
# $ & 1 ,
# $ & 1 , ,
# $ & 1 ,
#,$ +

( ) = {(φ ) 6 6 ( 6 4)} 7

# $ # $ 5 # $
#,$ +
_____________Download from JbigDeaL____________

' 6π =

# $ + # $ 1 !
# $ 1 ! #,$


' ! 6

# $ # $
# $ #,$

4 5 − + = ,
# $ 7
# $ # $ # $ G #,$ 4

5 ( + )×( + ) 16

# $ # $ # $
#,$ ? ,

, , ! 7
# $ 9 # $ # $ K
#,$ +

?
! , *3 # ! $ 7
# $ % *A ,? ? @ # $ % *3 ,? ? *
# $ 0 *3 ,? ? ? #,$ % *A ,? ? *

! ! , , ! , , !
/ ,* ! ,, ,, ,,
! 5 L × B"
# $ ,, 6 B, # $ ,, 6 ,
_____________Download from JbigDeaL____________

# $ ,, 6 B, #,$ ,, 6 ,

4 % ,; 9J - ,
0 ,"
% =<; ,
%<;
%7
8% M ;

;7
8; M %:
#%$:
# $ 3.
# $ ? , !
# $ A ?
#,$ +

5 - % , ! "
(.0F # 6 $:

%7 :
, )'+? #%7 $:

%7
8 #%$:
,:

%7
8
) ,#%$
- #%$
,
# $ # $
# $ & #,$ +

- 6 @6 ,? ! @
"
+ = 6 = 6 + + =
# $ 8 6@8 6 8 6?8 # $ 8 6@8 6 8 6?8
# $ 8 6@8 6 8 6?8 #,$ 8 6@8 6 8 6?8
_____________Download from JbigDeaL____________

B NNNNN 9 ,
C 0F , NNNNNNNN # $

I @
G
B
4

# $ O46 6 $6 # $ # 6 64P6 # $ O46 6 $6 #,$ O 6 64P6

D ,, , ! 1
, , ! , !
,, - "
6 6 54 6 5GG6 6 4 6 B 6 6 6 5 6 B 6 4 6
4D
# $ 4 # $ # $ D #,$

' ! 6ω ,ω 6
( )
4
− + =

# $ 9 6 K ω6 K ω # $ 6 9 ω6 9 ω
# $ 9 6 9 ω6 9 ω #,$ 9 6 K ω6 9 K ω

G
,# $7
# $7
# 6 $7

# ,# $6 ,# # # $$$$
# $ # $ # $ #,$

, , , !
, ,
→ { }
QQ
→ { }
QQ
→ { Q
4Q}
- 1111!JJ ! 1, ,
, , ! "
# $ 4 4 # $ 44 # $ 4 #,$ 44
_____________Download from JbigDeaL____________

, 0 , , !
! 8 ,
7 :
# 6 $
7#16!7 $:
7# 7 $
,
,
2 ! , , , 5 !
, ! * ,
!"
# $ # $ 5 # $ #,$

E !
(4 > 5 GB + > B+ > B + 4) 7

# $ # $ G # $ #,$

4 ,
= − + , = + − 7

# $ ( + + ) # $ ( + − ) # $ ( − − ) #,$ ( + − )
4 4 4 4

5 , ,
! , 7
5
# $ # $ # $ #,$
4

,
+ & ,
# $ + = 4( + ) # $ + = 4( + )
# $ + = − ( + ) #,$ +

B ' ! ! ,, , ! ,,
F0 RRRRRRRRRR ! ! J , RRRRRRR
, ! J
# $ 6 # $ 6
# $ 6 #,$ 6
JbigDeaL____________

D 3 9 , >
1
# $ , , > = 6 ><
# $ = 6 >< ,
# $ = 6 >< ,
#,$ = 6 ><

, ′′ + 4 ′ + =
− − − − −
# $ + # $ + 4
# $ + #,$ +

G ' ¬ ν 6
¬ ( )6 ¬ 6S
HE , 6

# $ # $ ,
# $ #,$ , ,

- , 36
= { >= }"
' →

'' ( + +
)
+ +
'''
# $ ' ! # $ ' ,'' # $ '' ,''' #,$ '' !

1 1 1 1
+ >( + )/ + > 7

# $ @ K ? K >)/
?K > # $ @ ? K >)/
? >KK
# $ >@ K ?/
)>? KK #,$ K >@ ?/
)> ? KK

- "
' 6

'' &
''' 1! T ( )
'H @ ! ,
# $ ' ,'' # $ '' ,''' # $ ' ,'H #,$ ' ,'''
JbigDeaL____________

4 1 ,
J

1 6J
18 18
?6 @6
@ @6 6
@6 ?6
? @6 6

' 6
7
# $ # $ # $ #,$

5 3 ={ 6 }6 = >
, = { > } ∪ ,
& !
# $ 6 # $ 6
# $ 6 #,$ 6

! 5L , J, 9 9
5 6 B5 , 2
,-(&? , ! ,, 7
# $ 65 # $ B6 5 # $ D6 #,$ 56 B

4 , A
! -6 %6 ; ,U 1
,@ * , G *6 !
, , !

6 @6 -6 %6 ; 7 , ! :
U 7 , 6# , !
! $

% 7
8 K@
; 7
8 # 9 $:
- 7
8 9@
U 7
8 >@
,:
_____________Download from JbigDeaL____________

5 # $ , 0 ,@ 9J
- A #46 $"
A # 6@7 $: :

@8
A 7
8
=@
A 7
8

A 7
8A # 9 6@$KA # 9 6@9 $
,:

# $ 0 , + × + #+< $
1 ) , ,
, ,

. %2'U 8 :
!
'+ &&8 !V 6. %2'U 6 . %2'U W :
0 , & +20(2 # 7'+ &&: + 7 $:

'6 I
6 .06 :

'7
8 +( M ,
I
78 +,

.078 V
'6I
W:
V
'6I
W78 V
I6'W
:
#I
6'P7
8 .0
,
,:

! C ,

I
I 4
I 4 4
I
I 5
_____________Download from JbigDeaL____________

I
I B
# $ ? ! @ ) ,)
# $ - "

B - ! 4 , , 9
, 6 @6 "? !

D # $ ? , B

# $ 1

! +

33# $
1 R1
1 # 1 $
4 1 H 1
5 1 R1
1 λ
B H ',H
D H # 1 $
H λ
G A 1 X

G # $ 1 > −( + ) ! 1

# $ , !
+
2 ! 33#'$

# $ F 0 , , !
,@
, #F6 H6 %6 ;7 $:

7 ,
6@7
,:
JbigDeaL____________

@7 ,
@6 7
,:

7
85:
@7
8H
,:
@,

7
8%:
@7
8;
,
,:
# $ ) , 1 0 7
< 6 @8 ,# $:

3 3 6 ≤ >
2 3 ,

#$ 3 +0 ,
# $ ) ! 6 1 ! 3
>
3 3 2 +0
,
G 6 4D6 6 6 6
F , , A
,

4 ( # $ C

( ∧ ) (¬ ∧ )
- ∧E
S ,6 SE ,¬

5 ' , , GB ! 5 ! 6
# $ , ! , 7
+ 7
( , , 7
' , , 75
' 7
_____________Download from JbigDeaL____________

+ 7
# $ , # $ 4B 6, ,
, , ! , ,
, , !

# $ ' 1
,+ +? = + +
# $ - @ 1 # , $
L "
@
?

B ! 7
.H' @6 ( *
.H' 6 *
3%' *6 ' 5 *
33 2F@
*3
2F@ .0.
&U
'+% *
? &@
I
+U 2F@
&
# $ - , ,"
# $ - ,@ !"
# $ - *3 1 "

D # $ !
'/
( ! ,
6 ! # ,, !$ ,
/ ,
#$ - , S E
"
# $ * ! , ,"
_____________Download from JbigDeaL____________

# $ , & , ! 1 , , ! ×
G × - J ,
&. , ! , # 1 $
,"

0 ,
# 9 KD$ ! E
? 1 6 @6
-(2 (.0#+7 $7 :

& .6 %0(+ + 7 :
@'+ &; 7 :

#+<89 $ ,#+=8KD$

+=
+ 78 :
@'+ &;7
8 :
%0(+ + 7
8 :
+=< ,

& .7
8+ , :
@'+ &;7
8@'+ &; K @> %0(+ + :
%0(+ + 7
8 %0(+ + > :
+7
8
,
-(2 (.07
8@'+ &;
,
,:

G ,
0 ,H ! J ? ,
2 2G

6 6 6 ,6 6 6 6 6 6 C
6 7 :

2 : H# $: H# $ ,:
0# $: 2 : H# $: H#,$ ,:
0# $: 25: H# $ ,:
JbigDeaL____________

0#,$: 2 : H# $ ,:
0# $: 0# $: 2D: H# $ ,:
0# $: 24:H# $:H# $ ,:
0# $: 2B: H# $ ,:
0# $: 0# $: 2 : H#C
$ ,:
0#C
$: 0#C
$: 0# $: 2G ,:
,
,:

, ,, , GG !
' )')( ,
6 D 6 D 6 46 B
- , , ) )2 #) ) 2 ,$ ,
22 )#2 2 ) $ , ") , ,

3 , A
# $ 2 ∩ A
# $ ' ∪ ! A"

* ! , "?
#- , , $

4
4 5
5

4 0 , ≥ 6 >

5 0 6 ,,, !

# $ ) , 4−5 +

# $ ? (≤ D ) , ! !
! 6 46 ,
JbigDeaL____________

B # $ , &# 6 @6 $
, , 7
6@ 6
2 & ,+ ) #4+)$ @ ! 9
, + ) #@ +)$
# $ ? ! &

D ,
F *(& # + . 6 '+2 ' F '(+6 ' ;6 A $
0F@3'2* & #0+ . 6 0 ' ;$
@((L # ' 36 + . 6 0+ . $
1 # $2 3 6 0&(I 6I
('+ ,
?'H'?
# $ A
# $ A ,
0+ . 8 S *+' 30F@3'2* &2E
# $ A , !
, . ,
_____________Download from

! "∪ "∪ ∩ "


" ∪ " ∪ " ∩ " ∩

# $ % &# ' ( # #)* ≤ $≤ + , -


. / $ ≤"
" ' " )
" 0 " . -

' 1 $ + - 2
- , 03 $ +

" = = 03 " = 03 =

" = 03 = 03 " . -

) 4
" 5
" 5
" 5
" 5 , 6

7 5 5
- (
#7
" " " "
'( ' '( '(

( 5 , - -
$8

"
( 8 + )+ ( 8 − ) "
( 8 + )− ( 8 − )
# #

"
( 8 + ) + # ( 8) + ( 8 − ) "
( 8 + ) − # ( 8) + ( 8 − )
# #
JbigDeaL____________

3 ,% × ,
× - $ × -
4
" 5 ,9
: -
" 2 ; 6 -

" 2 % 6 -
" 5 - - % 6 -
"%

< , - 4 ε
"
" 88" = ε > 8"
" 88"=
" 8=
-" 8 88"=
" " " " " " " " " " " -"

0 4
" 5 / 5
" ? ,
" @- , @ @#
@" % @#"
" @- @ @# @" %
@#"

8 ⊆ Σ= Σ %& * 4
" = { A A }
" = { ≥ }
" = { A A }
" = { ≥ ≥ }
4

" 88 = " = 8( )
88

" 28 < < =8 ( ) " # ≠ 8( )


_____________Download from

# B C
" D 15 BE1
" 2 2
151 2
151
2
15
"2 FGHGH1
FGHGH1
-" FGHGH1
" " " " " "
" " -" " " -"

' - , " -

" , 4
" 1 " ?
" . -

) 2 D ) -
IJ
4

" " ' " 3 "<

7 -
)4
" " " "

( K - -
" " !
" "

3 5 -
" L
"
"
-"
-
" # # " # # " # # " # # #
JbigDeaL____________

< 5 D < -

5HKP2
. 5H?
.H

KG..2
.@ KH ?+

OBEH?
"
" -
" -
"

0
"
" -
"
" M N

#8 , - 4
" L
"
" - -
" -

# KOP <
5 6 KOP
" #7( × ( " () E × < " ) E× ( " () E × (

## . KH5 <8<7
" " # " ' "7

#' Q -

" 8 8 8R 8 8 " 88888 R 888


" R " 8 R 8
_____________Download from JbigDeaL____________

#) D 2
SO -
4
" 2 - - -
" 2 - -
" 2 - -
" 2 -

#7 B

' #) #' 8

H, P

5 , #Q
5 6

" 8 " 87

" #−#' 87 " 87 ( # #' )

# K × × L -
- -

" −
( )= + −
" −
( )=( − + )
+ −
" −
( )= # #
" −
( )= #( − ) #( + )

## K K
∩ %φ 5 "
" K ,- -
" K -
" K -
" K ,-
_____________Download from

#' 4K ∧ .? ∨ OK T .O5

" (( → )∧ )→ " (( → )∧( ∧ )) →
" ( → ( ∨ψ ) ) " (( ∨ )↔( → ))

#) 4
" 5 L -

" 5 & # RR ! *

" 5 -

" ≠ φ @ ;@ =U
∈ = ∈

#7 . K ' U8

# '
+' # '
−'
" + = #
" + = #
' '

# '
+ # '

" + = #
" + = #
' '

θ − θ 8
#( 5
θ θ 8

" % θ% π "
" - " θ≠ θ

#3 5

) ) ) ' ) ' ) )
" × " × " × " ×
7# 7# 7# 7# 7# 7 7# 7

#< 2 # , K
, 4
" # " ∩ # " ∩K " ∪ #
_____________Download from

#0 ? , ≤ &8 * " % & S- ∈ ν


&&8 ** " , "
& S 8Q Q
5 "
" 8Q Q
"
" , 8Q Q
8Q
" . -

# 8 5
; U
; U ; U
C
%

"
C
%
" -
C
%
-
"
C
% C
%

"
C
% C
%

# 5 -
<0 0 )8 3 # 8 # 7 3 ( 0 38
" 8 " " # "'

# # 5
5 " %#

5 "%' +
)
/ 5 "
" O " " O "
'
" O ) " . -
_____________Download from

# ' 5 -

− +
" " " "
# # #

# ) C
78 7 (# 7 #8 7< 0 ' < '3 (8 #)
5 -

" ) 3" " 3 )" " < '" " ' <"

# 7 F
" #'R
" ! !# R #
B B# " "
- 5
" B ; B# " B U B# " B % B#
"

# ( 4
" P BKO P $
2
D HF $C $%8 R
P $>
H.?B
2
D .H $C $≠ O RR
OK? $C $"
H.?B
H.?P
" P BKO P# $
2
D HF $
P# $
H.?B
2
D .H $
OK? $>
H.?B
H.?P
" " " "
" " " " . -
_____________Download from

# 3 5

" - "
" #" @
B" K '" 1
?" )"

" !' !) B! ? # " !) !' B! ? #


" !) !' B!# ? " !' !) B!# ?

# < 888 E - 2
6 #88 E #(8 E - 5
E
" 7 " < " #' " 7)

# 0 ? M M -
"
"
"

" . -

# #8 D L , - 8>
B ? BMG ) <
- 5

" 8 " ) " < "0

## B D ## ' # 8
- ) ' # 8 ' # 8

VφQ
" /, -
" B? -
" - B B VφQ
" , # -
)

' # 8
_____________Download from

# ## B D # ##

B 8
B P ,
B #
B '

" + + " > >B


" ⊕ ⊕ " > B>B

# #' B D # #' 5
6
,
M 8

?8

8 B

B 8

? B

" ) " ' " # "

# #) - ,
E - D # #)
88 8 #8
B?
" +
88
" ( + + )( + + )
( )( )
8
" + +

" ( + )( + )
8

# #7 #7
-
P

" # " #7 " 8 " #


_____________Download from

'
#

( )= #
+ + ≤#
+ >#
D - W W
-

) )'
4H,

" ( 7# ) 3 )8 )'
" # ' 78 )8 (8 )'
" 8 (7 ' )< '3 )'
" < ( 7# ) ) )'
" 3 33 #3 (( < )'

7 B8 B 2
D

B B# D

8 8 +
8 >
8 ⊕
2 ) P , # H, - OK
# .? # OK 2-

( <8<7 88/ B

OK@ 8 88/
PN2 88/
$2 / 8 87/
MB/
/5
" φφ /4
" 1 - φφ /
3 - ( - 6
#7( E KG
"

3 ('

KG
D , -
88DD 8 8? 8DD 8

"
"
"

5H.

< D C
& # '* & # '* 2 ∈D
- T T '" %
" D - T
" D -

0 5 D { # ' } C

+# = + + ≥ W # = C = C
M - - -

8 = #
= #

# ## # ##

8
%2 = B? % H, ?

@ , @ % &1 B* & * M1"


M -
1 B
ε
X
ε
B
_____________Download from

ε " 5 @ - ,
@ ε
, "

# @- D #" P
P# 6 # -
" ? 6 #
P P#

" P "
6 #" ε

D "
8

1
P
8

1
B ? H

' F % #" " Y" δ Y φ"


-
- & *
5

" δ ( ) = {( )}
#" δ ( ) = {( )}
'" δ ( )={ }
)" δ( )={ }
7" δ ( # ) = {( # ε )}

(" δ ( # ) = {( # ε )}

3" δ ( # ε ) = {( # ε )}
_____________Download from JbigDeaL____________
www.estudentzone.com

) 9 :9 ! :
∀ ε − − < +
< +

D C
"5 9:9
L: %
L% RRRR
" 5 B O "
,
"
, "
- ′L
C W

" C% W L
C% 9:9
L:C
%,W
#" ,U , U"
'" 9 > :9
L:∩ 9::
L:
)" 9:9
L:C
% 9 > :9
L:W C% >
7"
("
3"
<"
0" 9:9
L:C
%

7 2 E K M B 1 . + 5 ZP 6
8
G
/ ," % ," ! V Q" > " 8
-
" 4
" ? 4

( @ - - , &8 R ! *
, V Q ? @
" -" X ! -X
" -" >-
_____________Download from

3 @ D )

)
( #
##

B
# #

? H

7
3φ φ
D

" O - ?L Q

" - H
" H4

< B ,
9: .U 5 G
4
- LC W ,C
C
% W 9 R .:
C W% W
C >L
" -#W
9 :; , C
%
L
C%
%," ∩%L
"W
9 :% ,"
V, Q"

V,Q Q"
W

0 B M ,

" 4
" - 4
H, "
- C W
#W
C% 8
C "W
C
7W
V CV "W
#W
W V CV "W
W
=H, ="
C
%'W : "W
V 'CV "

#8 B , 1H51" C
H H>H& I
>J*
H H =H & IJ
*
H & *
H H"
K! ,
?

> "= > " 1?51 - -

# 5 L C
; U ,
L ;- U 6 - "
- 8
1 1 1# 1' 1) 17

.% #
P %#
'
)
1
CL .
1'
#CL P
17
'C
1#

)C
1)
_____________Download from

#
,C

## Q 2
M8 M M#
K8 K K#

P , . B -
K8 K K# K8 K K# K8 K K#
M8 ) # M8 8 # # # 8
M 7 M 8 '
M# # ' M# 8 #

" 1
" M8
K4

#' -
6 )E 5 C
? 8C D 5 <
,

? C ? '# C
? #C ? W
? 'C ? #W
" , 4
" , 6 4

#) B D 7

? F ?# F# ?' F'

BE BE BE
B

D 7

" ? G
- -
_____________Download from

8 8 8 18
8 8 1

13
" @- 1)

#7 78 P/6 ?P
?P 888
?P
788 5
#888 E S - 6 )E
-
88[ 4

- B " - B "
% 78 S % #88 S
16 / 16 /
<E 8 <8 )E 8 0<
(E 8 08 (E 8 00
() E 8 07 () E 8 007

16 /
#78 P 8

#( -
C

- B " - # " - '

% 78 S % #88 S %7 \ S

16 / 16 / 16 /
<P 8 <8 )P 8 0< #(8 P 8
(P 8 08 (P 8 00
_____________Download from JbigDeaL____________

() P 8 07 () P 8 007

" 6 - # -
- 88 4
" - - 6 -
- # 4

#3
G1HK1 G ] G . / 5 "
OOE1 ] 5 . "
2
11GH? ] G ] ? "
H, H

"σG ]%( G ] 5 G1HK1 2


11GH?" OOE1""
" σ . OOE1 σ / 5 " %? G1HK1 2
11GH?"""
Hosted at www.educationobserver.com
_____________Download from
For more papers visit www.educationobserver.com/forum

!
" #
$
% & ' % & (! % & '! % & )

( * + + (
−( =
,
+ + -
% & . % & . (

% & . ( % & .

' + /
" −0
( ) "
1
) 0

% & % & )0 % & % & ()

) + + % & +
+ + $
% & 2 % & 3
% & 4 % & 5 +

! + +

%5& 5 % &6
%7& 8 2 %(& 3 9
%4& : %'& 3 ;
%3& 4 4 %)& 3 . 4

% & 5 - ( 7- ) 4 - 3 ' % & 5 - ' 7- ) 4 - 3 (


% & 5 - ' 7- ) 4 - ( 3 % & 5 - ) 7- 4- ( 3 '
_____________Download from JbigDeaL____________

" , +

11= ⊕

11= <

11=
+ + $
% & =⊕> + . =<> .
% & 7 =⊕> =<> + .
% & =⊕> . =<> + .
% & + .

# + + + . +/ / +/
+ ++ $
% & 5 % & 5
% & 5
% & 5

0 5 ? + @
, ++ + +
+ $
% & 5 + ?
% & 5 + ?A
% & 5 + ?
% & + .

B / + + .
% & / + /
% & / . +
/ + /
% &
% & / . +
+ /

C +
% &
% &
% &
% & + .
_____________Download from JbigDeaL____________

( !

( ? < + /< D E ? @D /< . + @</


% & E % & / % & % &

(( *2 #! 0 0! / . +
.
% & C % & #!C % & '4C % & ')C

(' ; + *2 ('(
% & @ . + . .
% & @ . + . +
% &
% & @ . + .

() + +
%5& 3:5 ,
F % &C *5:
%7& 4 %(& 3
%4& , ,
F %'& ;
%3& 4 4 * %)& 5?G
% & 5 - ) 7- ' 4 - 3 ( % & 5 - ( 7- 4- ' 3 )
% & 5 - ) 7- ' 4 - ( 3 % & 5 - ( 7- ' 4 - ) 3

(! 5 +) ,
4 #) 0 %)
5?G& #) 0( %) &
+
% & % &
% & % & + .

'

' ? %H <& H +
< + < D / % & + +
+ %H <&$
% & %H <& % & %H <& 5
% & %H <& % & + .
_____________Download from JbigDeaL____________

'( + + $
% & % ∨ & % & ∨% & % & ∨% & % & % &

'' , + C . + +
= > .$
% & (
% & '
% &
% &
+

') 6. ΣDI J + + $
% & 2 + . Σ
% & 2 + . Σ
% & 2 + . Σ
% & 2 + . Σ

'! ? + + +
% & . +
% & + + +
% & + /
% & +

'" + + 1

%5& 3 % &*
%7& 7 %(& 245
%4& %'& ?,
9
%3& , %)& 9,
9

% & 5 - ' 7- ) 4 - ( 3 % & 5 - ) 7- ' 4 - ( 3


% & 5 - ( 7- ) 4 - 3 ' % & 5 - ' 7- ) 4 - ' 3 (

'# ,
F
% & + +
% & / + + +
% & (
% & + .
_____________Download from JbigDeaL____________

'0
% &
% & + +
% & / + +

% & + = >

'B
% & ,
F
% & +
% & / . ,
F
% & . +

' 3 +
% & . .
% & ?*G +
% &
% & + .

) (

) / . + + ( )=( (
−( +" .
K (L
$
% & " % & % & ( % &! !

)( ? ( )
D / ( /
+ + , + /
( +
% &
% & +5
% & +5
% & @

)' G + . ′′ % & D+% & % & ′′ % &D @


+ + . + + + 1
% & +% & %+% & E +% && %+% & E +% & D +%( &&
( (
% & +% & ( +% &E+% &
_____________Download from JbigDeaL____________

( (
% & +% & ' +% &
( (
% & +% & E +% & +% &E +% &E +% &

)) 5 %/& % & D! % & D) %(& D B %'& D (


.
% & % & ( % & ' % &)

)! 5 . ( ' ) ! " #0 .
. + +
. $
% & !' ( ) #0 " % & !' (")0#
% & !'() " #0 % & !' ( ) #" 0

)" ? % & + + % & D % & D ( E + ≥(


(
+ + $
% & ( )= % & % &D % &

% & % &D % & % & + .

)# 5 8 ;G2C %4&
, 2 * %8 4 M& M
;; 3? :, %8& 9 +

% & % &
% & % &

)0 6. + ;
; 5A
/ 1 A
; 7A
/ @1
2
7A
; 4A
1 A
2(
4A
5A
_____________Download from

. 2 2(
% & / 5 / +7 @ 2
/ +7 2(
% & / 7 @ 2
/ +7 @ 2(
% & / 7@ 2
/ +5 2(
% & + .

)B / % < &< N
= > + =E> =<> =↑>% / & . 4;G
. +% < & +
% & = > >
E> =<> % & = ↑> =<>
> > % & = ↑> =E>
> >
% & .

) @ ( )

− (
/ ′′ ( ) ∈
(
+ @ + @
≤ )
# =

% & " % & % & " % &

! ! (

! ? ( )= + + + + +
. $
% & / ++ % & /@ ++
% & /@ ++ % & ++

!( 4 + 5 + / + + 0 0!

:O,
5 !! C
:O,4 (! C
5334
355
% & #5C % & 0 C % & ! C % & ((C
_____________Download from JbigDeaL____________

!' 5 +
% & + +
% & / +
+
% & B % & !
% & 0 % & + .

!) 6. (() ) = ')

. + / 1
% & % & 0 % & ! % &"

!! 4 9 + ( %
+ + & 1
( ) = Σ0 B
+
( ( ) = Σ# 0 ( ' ) ! +
+(
( ) = Σ0 B
+'D$
+ '

% & ΣB % & ΣB % & Σ 0 B % & Σ0 !

" (

" 5 ≤ + ={ ( } ≤ +
≤ + ≥ + 2

% & P % & E( % & % &

"( ? 6 . . Q
Q + − =0 − = ( + 6

% & 0 % & ) % & ( % & (!

"' + . + I ( ' )J
% & ! % & " % & () % &)

") + + / . I J +
$
% & <% E &< % & < < % & << % & % E &<
_____________Download from JbigDeaL____________

"! + + $
% & 6. : :

% & . + .
% & ? . +
% & ? / +

"" + + . I J
$

% & { ⊂ ∈{ } <} % & { ∈{ } <}


% & { ≥ }
% & { . { }}
*
1 . = >

"# 5 ' ( +
* + + .
% & ' % & ! % & ) % &"

"0 ; D NNB +

;% /&
I J
.% /&
+ .
+ / .% /& + ;% /&
+
$
% & % & ( % & '
% & + .

"B 9 *% & +
. + +
+ + 1

% & + + +
% & + +
_____________Download from JbigDeaL____________

% & +
% & + .

" ? * % & 2% +& + + 2


+ + * 4 + + *
2
% & , * % & , 2
% & 3 + * % & 3 + 2
+ + + .$
% & +% & % & % & % & .
% & 5 +% & % & % & % & .
% & 7 % & % & .
% & 7 % & % & .

# 5 3+ + 0 0!
+ 99 / 3 3' . .
+ + 4;G + + + +
% & G 5 3 %+ + & 3+ +
+ % .
&
% & . + + +
99 + . #

0 ? ?D I ( NN & ≥ ;
D
/ 1↑ A. 1

+ + ( ( − ≤≤
(
4 + + /
+ /
+ % 1↑ &1 ↑ A
( . 1↑ A
'
) + D 1D
! + ↑ / D 1
"
# 1 ↑ /A
0 1D A
B 1 ↑ /A
1D A
1D %&
(
'
_____________Download from JbigDeaL____________

B 4 .
%/ & ≤/≤ ≤ ≤ ≥( .
( ) ( ( ( ) Q − ( ≤ − ( ≤ +

{( ) ( ( ( )} ( − ( )( + ( − ( )(
% & + $
/
% & + / $
/ $

4 + ; ; /
1
. @1
%/&1
+/≤)
/1 / E @A
%/&A
@1D/E

%< <&
%< <&
@D A
%@&A
%@&

% & 2 = > .
%& . $
% &C = > $
% & . +
+ $

4
2 2
2 ;8*
; ;
; ε
8 8
8 ε
* *
* ε
_____________Download from JbigDeaL____________

2 ;8 * 2 A
=ε>
+ + + Q ≤
% & . + Q $
% & 9

5 + 5

( 4 / % . + &
. +
. M
.
% & + M
$
% & + M
$
% & + M $

' ? 9 + + + I (N J
I (N J ≥ ≥
% & C + + 9$
% & C + + 9 + +% &D + ≤≤ $
% & C + + 9 + +% &R+%Q& + ≤ ≤Q≤ $

) ? * + /. . 5 +

= {( )( )∈ ( )∈ }
% & ; . . 5
% & 3+ ≤ . + ≤ ( ∃
∈ ∈ ( % &∈* ; . ≤

! 4 + +
9 9% 1 &1 A

,
+ % RD & % RD & 91D

919% & E 9% - &A


A
_____________Download from JbigDeaL____________

− −
G = + +

5 .
/
% & . + 9% (&$
% & . + 9% &$
% & C . + 9
. 9% &

" 5 @ + . ++
+ +
+ + + +
+ / + + +

% & ; . @ +
(
% & , @ + ≤
- + $ /

# 5 5 ≥ . 5K L5K
(LN 5K L
+ + + +
. + 5 5K
,L5KE LNN 5KQL + .
≥: . . , = E >+ /
4 + / ,
/ + /

( 1E A
Q1D A
' 2 1D A
) 1 A+ 1D + A
! +
" ,
+
# +QD + 1D
0
B
Q1DQE A
1D
(
'
)
! ,
+%Q - ,
&R 1DQ - A
" 1D - 5KL
A
# 1D E A
_____________Download from JbigDeaL____________

0
B % E &A
(

0 4 + + =4> + . +
+
I
. A
< /A
IA
+ A
< . % < <Q&
I
< < (A
<QD A
D A
+% PD G??& ( D /A
1
<Q D A
+ % ( DD G??& A
( PD G??&
I
+% . PD ( . & %2 &
I
%<Q&EEA / D (A D (1 %2(&
J
(D ( /A
J
/ D G??A
A
J
5 % - (& +
% & C 2 $
% & / +
2( / $
% & + + . Q
+ $
_____________Download from JbigDeaL____________

B 5 7E + (
. 5 : . :E %+
& ( . + +
+ + 5 .
+ + + + +

% & + . + 7E
. % ≥ (&$
% & / + 7E + )
!( . $
% & + . 7E +
% ≥ &$

( 4 + +
. % & + > .
+ > .

( 6. ? +
+
= { ′∈ }
S

= { . ? }

(( 5 ? +; + 1
% & / . . =E> =R>
% & 5
% & = >
% & 4 /= N >
6. +?

(' ? + + +
/
11 1=
11= +

11= <

11= ( )
( ) 11
, . / + ? +/
+ %E & %E & %< & %< & %1D &
%1D &
_____________Download from JbigDeaL____________

/ . /
+/ + T +
. + > . % & .
% &< .

() 4 + + ; 1
; : A
. U1 A
5A
. T1 A
7A
. H1 A
4A
. H1 A
%< 4<&
1
%< 4<&
%< 7<&
1
4A%< 4<&
5A%< 5<&
1
%< 7<&
%< 5<&
1
7A%< 7<&
1
%< 5<&
Hosted at www.educationobserver.com
%<: <&
For more papers visit www.educationobserver.com/forum
Hosted at www.educationobserver.com

For more papers visit www.educationobserver.com/forum

SECTION - A
1. This question consists of 35 THIRTY-FIVE multiple questions of ONE mark each.
For each question, four possible alternatives (A, B, C and D) are given, out of
which ONLY ONE is correct. Indicate the correct answer in the boxes
corresponding to the questions only on the FIRST sheet of the answer book.

1.1 A die is rolled three times. The probability that exactly one odd number turns up
among the three outcomes is
1 3 1 1
(a) (b) (c) (d)
6 8 8 2

1.2 Consider the following set of equations:


x + 2y = 5
4 x + 8y = 12
3x + 6y + 3z = 15
This set
(a) has unique solution (b) has no solutions
(c) has finite number of solutions (d) has infinite number of solutions

1.3 Which of the following statements applies to the bisection method used for
finding roots of functions:
(a) converges within a few iterations
(b) guaranteed to work for all continuous functions
(c) is faster than the Newton-Raphson method
(d) requires that there be no error in determining the sign of the function.

1.4 Consider the function y = x in the interval [-1,1]. In this interval, the function is

(a) continuous and differentiable


(b) continuous but not differentiable
(c) differentiable but not continuous
(d) neither continuous nor differentiable

1.5 What is the converse of the following assertion?


I stay only if you go
(a) I stay if you go
(b) If I stay then you go
(c) If you do not go then I do not stay
(d) If I do not stay then you go
1.6 Suppose A is a finite set with n elements. The number of elements in the largest
equivalence relation of A is
(a) n (b) n2 (c) 1 (d) n + 1

1.7 Let R1 and R2 be two equivalence relations on a set. Consider the following
assertions:
(i) R1 ∪ R2 is an equivalence relation
(ii) R1 ∩ R2 is an equivalence relation
Which of the following is correct?
(a) Both assertions are true
(b) Assertion (i) is true but assertion (ii) is not true
(c) Assertion (ii) is true but assertion (i) is not true
(d) Neither (i) nor (ii) is true

1.8 The number of functions from an m element set to an n element set is


(a) m + n (b) mn (c) nm (d) m*n

1.9 If the regular set A is represented by A = (01 + 1)* and the regular set ‘B’ is
represented by B = ((01)*1*)*, which of the following is true?
(a) A ⊂ B (b) B ⊂ A
(c) A and B are incomparable (d) A = B

1.10 Which of the following set can be recognized by a Deterministic Finite state
Automaton?
(a) The numbers 1, 2, 4, 8, ……………. 2n , ………… written in binary
(b) The numbers 1, 2, 4, ………………., 2n , …………..written in unary
(c) The set of binary string in which the number of zeros is the same as the
number of ones.
(d) The set {1, 101, 11011, 1110111, ………..}

1.11 Regarding the power of recognition of languages, which of the following


statements is false?
(a) The non-deterministic finite-state automata are equivalent to deterministic
finite-state automata.
(b) Non-deterministic Push-down automata are equivalent to deterministic Push-
down automata.
(c) Non-deterministic Turing machines are equivalent to deterministic Push-down
automata.
(d) Non-deterministic Turing machines are equivalent to deterministic Turing
machines.
(e) Multi-tape Turing machines are equivalent to Single-tape Turing machines.
1.12 The string 1101 does not belong to the set represented by
(a) 110*(0 + 1) (b) 1 ( 0 + 1)* 101
(c) (10)* (01)* (00 + 11)* (d) (00 + (11)*0)*

1.13 What happens when a bit-string is XORed with itself n-times as shown:
B ⊕ B ⊕ B ⊕ ( B K n times 
( (
 
(a) complements when n is even (b) complements when n is odd
(c) divides by 2n always
(d) remains unchanged when n is even

1.14 A multiplexor with a 4 bit data select input is a


(a) 4:1 multiplexor (b) 2:1 multiplexor
(c) 16:1 multiplexor (d) 8:1 multiplexor

1.15 The threshold level for logic 1 in the TTL family is


(a) any voltage above 2.5 V
(b) any voltage between 0.8 V and 5.0 V
(c) any voltage below 5.0 V
(d) any voltage below Vcc but above 2.8 V

1.16 In serial communication employing 8 data bits, a parity bit and 2 stop bits, the
minimum band rate required to sustain a transfer rate of 300 characters per
second is
(a) 2400 band (b) 19200 band
(c) 4800 band (d) 1200 band

1.17 The octal representation of an integer is 3428. If this were to be treated as an


eight-bit integer is an 8085 based computer, its decimal equivalent is
(a) 226 (b) -98 (c) 76 (d) -30

1.18 Which of the following devices should get higher priority in assigning interrupts?
(a) Hard disk (b) Printer
(c) Keyboard (d) Floppy disk

1.19 Which of the following addressing modes permits relocation without any change
whatsoever in the code?
(a) Indirect addressing (b) Indexed addressing
(c) Base register addressing (d) PC relative addressing
1.20 Which of the following is true?
(a) Unless enabled, a CPU will not be able to process interrupts.
(b) Loop instructions cannot be interrupted till they complete.
(c) A processor checks for interrupts before executing a new instruction.
(d) Only level triggered interrupts are possible on microprocessors

1.21 Which one of the following algorithm design techniques is used in finding all pairs
of shortest distances in a graph?
(a) Dynamic programming (b) Backtracking
(c) Greedy (d) Divide and Conquer

1.22 Give the correct matching for the following pairs:

(A) O (log n) (P) Selection


(B) O (n) (Q) Insertion sort
(C) O (n log n) (R) Binary search

(D) O n2 ( ) (S) Merge sort

(a) A – R B – P C – Q D - S (b) A – R B – P C – S D - Q
(c) A – P B – R C – S D - Q (d) A – P B – S C – R D - Q

1.23 How many sub strings of different lengths (non-zero) can be found formed from a
character string of length n?
n ( n + 1)
(a) n (b) n2 (c) 2n (d)
2

1.24 Which of the following statements is false?


(a) A tree with a n nodes has (n – 1) edges
(b) A labeled rooted binary tree can be uniquely constructed given its postorder
and preorder traversal results.
(c) A complete binary tree with n internal nodes has (n + 1) leaves.

(
(d) The maximum number of nodes in a binary tree of height h is 2h+1 − 1 )
1.25 In a resident – OS computer, which of the following systems must reside in the
main memory under all situations?
(a) Assembler (b) Linker
(c) Loader (d) Compiler
1.26 Which of the following statements is true?
(a) SLR parser is more powerful than LALR
(b) LALR parser is more powerful than Canonical LR parser
(c) Canonical LR parser is more powerful than LALR parser.
(d) The parsers SLR, Canonical CR, and LALR have the same power

1.27 Type checking is normally done during


(a) lexical analysis (b) syntax analysis
(c) syntax directed translation (d) code optimization

1.28 A linker reads four modules whose lengths are 200, 800, 600 and 500 words,
respectively. If they are loaded in that order, what are the relocation constants?
(a) 0, 200, 500, 600 (b) 0, 200, 1000, 1600
(c) 200, 500, 600, 800 (d) 200, 700, 1300, 2100

1.29 Which of the following is an example of a spooled device?


(a) The terminal used to enter the input data for the C program being executed
(b) An output device used to print the output of a number of jobs.
(c) The secondary memory device in a virtual storage system
(d) The swapping area on a disk used by the swapper.

1.30 When the result of a computation depends on the speed of the processes
involved there is said to be
(a) cycle steating (b) rare condition
(c) a time lock (d) a deadlock

1.31 A counting semaphore was initialized to 10. Then 6 P (wait) operations and 4V
(signal) operations were completed on this semaphore. The resulting value of the
semaphore is
(a) 0 (b) 8 (c) 10 (d) 12

1.32 A computer has six tape drives, with n processes competing for them. Each
process may need two drives. What is the maximum value of n for the system to
be deadlock free?
(a) 6 (b) 5 (c) 4 (d) 3

1.33 Given two union compatible relations R1(A,B) and R2 (C,D), what is the result of
the operation R1A = CAB = DR2?
(a) R1 ∪ R2 (b) R1 × R2 (c) R1 − R2 (d) R1 ∩ R2
1.34 Which normal form is considered adequate for normal relational database design?
(a) 2 NF (b) 5 NF (c) 4 NF (d) 3 NF

1.35 There are five records in a database.

Name Age Occupation Category


Rama 27 CON A
Abdul 22 ENG A
Jeniffer 28 DOC B
Maya 32 SER D
Dev 24 MUS C

There is an index file associated with this and it contains the values 1,3,2,5 and
4.Which one of the fields is the index built from?
(a) Age (b) Name (c) Occupation (d) Category

2. This question consists of 20 (TWENTY) multiple-choice questions (2.1 – 2.20) of


TWO marks each. The answers to the multiple choice questions of this section
MUST be written only in the boxes corresponding to the questions, in the second
page of the answer book.

2.1 The rank of the matrix given below is:

1 4 8 7
0 0 3 0
4 2 3 1
3 12 24 2
(a) 3 (b) 1 (c) 2 (d) 4

1 a bc
2.2. Consider the following determinant ∆ = 1 b ca .
1 C ab

Which of the following is a factor of ∆ ?


(a) a+b (b) a-b (c) a+b+c (d) abc
2.3. The binary relation R = {(1,1)} , (2,1) , (2, 2) , (2, 3) , (2, 4) , (3,1) , (3,2) , (3, 3) , (3, 4) on
the set A = {1,2,3,4} is
(a) reflexive, symmetric and transitive
(b) neither reflexive, nor irreflexive but transitive
(c) irreflexive, symmetric and transitive
(d) irreflexive and antisymmetric

2.4. In a room containing 28 people, there are 18 people who speak English, 15
people who speak Hindi and 22 people who speak Kannada. 9 persons speak both
English and Hindi, 11 persons speak both Hindi and Kannada whereas 13 persons
speak both Kannada and English. How many people speak all three languages?
(a) 9 (b) 8 (c) 7 (d) 6

2.5. Let L be the set of all binary strings whose last two symbols are the same. The
number of states in the minimum state deterministic finite 0 state automaton
accepting L is
(a) 2 (b) 5 (c) 8 (d) 3

2.6. Which of the following statements is false?


(a) Every finite subset of a non-regular set is regular
(b) Every subset of a regular set is regular
(c) Every finite subset of a regular set is regular
(d) The intersection of two regular sets is regular

2.7. The function represented by the Karnaugh map given below is


BC
A 00 01 10 11
0 1 1 0 1

1 1 0 0 1

(a) A.B (b) AB+BC+CA (c) B ⊕ C (d) A.BC

2.8. Which of the following operations is commutative but not associative?


(a) AND (b) OR (c) NAND (d) EXOR

2.9. Formatting for a floppy disk refers to


(a) arranging the data on the disk in contiguous fashion
(b) writing the directory
(c) erasing the system area
(d) writing identification information on all tracks and sectors
2.10. The address space of 8086 CPU is
(a) one Megabyte (b) 256 Kilobytes
(c) 1 K Megabytes (d) 64 Kilobytes

2.11. A complete n-ary tree is one in which every node has O or n sons. If x is the
number of internal nodes of a complete n-ary tree, the number of leaves in it is
given by
(a) x(n – 1) +1 (b) xn - 1 (c) xn + 1 (d) x(n+1)

2.12. What value would the following function return for the input x = 95?
Function fun (x:integer):integer;
Begin
If x > 100 then fun : x – 10
Else fun : fun(fun (x + 11))
End;
(a) 89 (b) 90 (c) 91 (d) 92

2.13. What is the result of the following program?


program side-effect (input, output);
var x, result: integer:
fucntion f (var x:integer):integer;
begin
x:x+1;f:=x;
end
begin
x:=5
result:=f(x)*f(x)
writeln(result)
end

(a) 5 (b) 25 (c) 36 (d) 42

2.14. Let A be a two-dimensional array declared as follows:


A: array [1 …. 10] [1 …… 15] of integer;
Assuming that each integer takes one memory locations the array is stored in
row-major order and the first element of the array is stored at location 100, what
is the address of the element A[i][j]?
(a) 15i + j + 84 (b) 15j + i + 84
(c) 10i + j + 89 (d) 10j + i + 89
2.15. Faster access to non-local variables is achieved using an array of pointers to
activation records called a
(a) stack (b) heap
(c) display (d) activation tree

2.16. The overlay tree for a program is as shown below:

Root 2 KB

A 4 KB B 6 KB C 8 KB

D 6 KB E 8 KB F 2 KB G 4 KB

What will be the size of the partition (in physical memory) required to load (and
run) this program?
(a) 12 KB (b) 14 KB (c) 10 KB (d) 8 KB

2.17. Consider n processes sharing the CPU in a round-robin fashion. Assuming that
each process switch takes s seconds, what must be the quantum size q such that
the overhead resulting from process switching is minimized but at the same time
each process is guaranteed to get its turn at the CPU at least every t seconds?
t − ns t − ns t − ns t − ns
(a) q ≤ (b) q ≥ (c) q ≤ (d) q ≥
n −1 n −1 n +1 n +1

2.18. If an instruction takes i microseconds and a page fault takes an additional j


microseconds, the effective instruction time if on the average a page fault occurs
every k instruction is:
j i+ j
(a) i + (b) i + j * k (c) (d) ( i + j ) * k
k k

2.19. Which of the following query transformations (i.e. replacing the l.h.s. expression
by the r.h.s. expression) is incorrect? R1 and R2 are relations, C1, C2 are selection
conditions and A1, A2 are attributes of R1?

( ) (
(a) σ c1 σ c1 ( R1 ) → σ c 2 σ c 2 ( R1 ) ) ( ) (
(b) σ c1 π A1 ( R1 ) → π A1 σ c1 ( R1 ) )
(c) σ c1 ( R1 ∪ R2 ) → σ c1 ( R1 ) ∪ σ c1 ( R2 ) (d) π A1 (σ c1 ( R1 ) ) → σ c1 (π A1 ( R1 ) )
2.20. Suppose the domain set of an attribute consists of signed four digit numbers.
What is the percentage of reduction in storage space of this attribute if it is
stored as an integer rather than in character form?
(a) 80% (b) 20% (c) 60% (d) 40%

3. (a) Two friends agree to meet at a park with the following conditions. Each will
reach the park between 4.0 p.m. and 5.00 p.m. and will see if the other has
already arrived. If not, they will wait for 10 minutes or the end of the hour
whichever is earlier and leave. What is the probability that the two will not
meet?
(b) Give a regular expression for the set of binary strings where 0 every is
immediately followed by exactly k 1’s and preceded by at least k 1’s (k is a
fixed integer)

4. Design a deterministic finite state automaton (using minimum number of states)


that recognizes the following language:
L = {w ∈ {0, 1}*|w interpreted as binary number (ignoring the leading zeros) is
divisible by five.

5. (a) The implication gate, shown below has two inputs (x and y); the output is 1
except when x = 1 and y = 0, realize f = xy + xy using only four implication
gates.
(b) show that the implication gate is functionally complete.

6. (a) Solve the following recurrence relation


xn = 2 xn −1 − 1, n > 1
x1 = 2
(b) Consider the grammar
S  Aa|b
A  Ac|Sd|∈
Construct an equivalent grammar with no left recursion and with minimum
number of production sales.

7. (a) Suppose we have a database consisting of the following three relations.


FREQUENTS (student, parlor) giving the parlors each student visits.
SERVES (parlor, ice-cream) indicating what kind of ice-creams each parlor
serves.
LIKES (student, ice-cream) indicating what ice-creams each student likes.
(Assume that each student likes at least one ice-cream and frequents at least
one parlor)
Express the following in SQL:
Print the students that frequent at least one parlor that serves some ice-
cream that they like.
(b) In a computer system where the ‘best-fit’ algorithm is used for allocating
‘jobs’ to ‘memory partitions’, the following situation was encountered:

Partitions sizes in KB 4K 8K 20K 2K


Jobs sizes in KB 2K 14K 3K 6K 6K 10K 20K 2K
Time for execution 4 10 2 1 4 1 8 6

When will the 20K job complete?

SECTION – B
This section consists of TWENTY questions numbered 8 to 27 of FIVE marks each.
Attempt ANY TEN questions. Answers must be given in the answer book provided.
Answer for each question must start on a fresh page and must appear at one place only.
(Answers to all parts of a question must appear together).
8. (a) Find the points of local maxima and minima, if any, of the following function
defined in 0≤x≤6.
x 3 − 6 x 2 + 9 x + 15
(b) Integrate
π
∫ π x cos xdx

9. Derive the expression for the number of operations required to solve a system of
linear equations in n unknowns using the Gaussian Elimination Method. Assume
that one operation refers to a multiplication followed by an addition.

10. (a) Prove by induction that the expression for the number of diagonals in a
n ( n − 3)
polygon of n sides is
2
(b) Let R be a binary relation on A = {a, b, c, d, e, f, g, h} represented by the
following two component digraph. Find the smallest integers m and n such
that m < n and Rm = Rn.
e
b

d
f

c h g
11. Suppose A = {a,b,c,d} and Π1 is the following partition of A
Π1 ={{a,b,c}{d}}
(a) List the ordered pairs of the equivalence relations induced by Π1 .
(b) Draw the graph of the above equivalence relation.
(c) Let Π2 = {{a} , {b} , {C} , {d}}
Π3 = {{a, b, c, d}}

and Π 4 = {{a, b}{c, d}}

Draw a Poset diagram of the poset,


{Π1 , Π2 , Π3 , Π 4 } , refines

12. Let (A, *) be a semigroup, Furthermore, for every a and b in A, if a ≠ b, then a*b,
b ≠ b*a.
(a) Show that for every a in A
a*a =a
(b) Show that for every a, b in A
a*b*a =a
(c) Show that for every a, b, c in A
a*b*c = a*c

13. Let M = ({q0 , q1} , {0,1} , {z0 , X } , δ , q0 , z0 , φ ) be a Pushdown automation where δ is


given by
δ ( q0 ,1, z0 ) = {( q0 , xz0 )}
δ ( q0 , ∈, z0 ) = {( q0 , ∈)}
δ ( q0 ,1, X ) = {( q0 , XX )}
δ ( q1 ,1, X ) = {( q1 , ∈)}
δ ( q0 , 0, X ) = {( q1 , X )}
δ ( q0 , 0, z0 ) = {( q0 , z0 )}

(a) What is the language accepted by this PDA by empty store?


(b) Describe informally the working of the PDA.

14. (a) Let G1 = (N, T, P, S1) be a CFG where,


N = {S1, A, B} T = {a, b} and
P is given by
S1  a, S1 b S1  a B b
S1  a A b B  Bb
AaA Bb
Aa
What is L(G1)?
(b) Use the grammar in Part (a) to give a CFG

for ( )
L2 = ai bi ak b1 i, j, k.1 ≥ 1, i = j or k = 1 by adding not more than 5
production rules.
(c) Is L2 inherently ambiguous?

15. (a) Draw the schematic of 8085 based system that can be used to measure the
width of a pulse. Assume that the pulse is given as a TTL compatible signal
by the source, which generates it.
(b) Write the 8085 Assembly Language program to measure the width of the
pulse. State all your assumptions clearly.

16. Design a synchronous counter to go through the following states:


1, 4, 2, 3, 1, 4, 2, 3, 1, 4 ……………

17. Calculate the total time required to read 35 sectors on a 2-sided floppy disk.
Assume that each track has 8 sectors and the track-to-track step time is 8
milliseconds. The first sector to be read is sector 3 on track 10. Assume that the
diskette is soft sectored and the controller has a 1-sector buffer. The diskette
spins at 300 RPM and initially; the head is on track 10.

18. For a set-associative Cache Organization, the parameters are as follows:


tc -- Cache access time
tm -- Main memory access time
l -- number of sets
b -- block size
k*b -- set size
Calculate the hit ratio for a loop executed 100 times where the size of the loop is
n * b, and n = k*m is a non-zero integer and 1 < m ≤ 1.
Give the value of the hit ratio for 1 = 1
19. (a) Let p be a pointer as shown in the figure in a single linked list.

…………. ………….
p cell i cell(i+1) cell(i+2) cell(i+3)

What do the following assignment statements achieve?


q: = p  next
p  next:=q  next
p  next:=(q  next)  next
(p  next)  next:=q
(b) Compute the post fix equivalent of the following expression.
a
3 * log ( x + 1) −
2

20. Draw the binary tree with node labels a, b, c, d, e, f and g for which the inorder
and postorder traversals result in the following sequences.
Inorder afbcdge
Postorder afcgedb

21. (a) Derive a recurrence relation for the size of the smallest AVL tree with height
h.
(b) What is the size of the smallest AVL tree with height 8?
(c)

22. (a) An identifier in a programming language consists of up to six letters and


digits of which the first character must be a letter. Derive a regular
expression for the identifier.
(b) Build an LL (1) parsing table for the language defined by the LL(1) grammar
with productions
Program  begin d semi X end
X  d semi X | sY
Y  semi s Y | ∈

23. Let the attribute ‘val’ give the value of a binary number generated by S in the
following grammar:
S  L.L | L
L  LB | B
B0|1
For example, an input 101.101 give S.val = 5.625
Construct a syntax directed translation scheme using only synthesized attributes,
to determine S.val.
Hosted at www.educationobserver.com For more papers visit www.educationobserver.com/forum

24. (a) Four jobs are waiting to be run. Their expected run times are 6, 3, 5 and x.
in what order should they be run to minimize the average response time?
(b) Write a concurrent program using par begin-par end to represent the
procedure graph shown below.
S1

S2 S3

S4
S5

25. (a) Free disk space can be kept track of using a free list or a bit map. Disk
addresses require d bits. For a disk with B blocks, F of which are free, state
the condition under which the free list uses less space than the bit map.
(b) Consider a disk with C cylinders, t tracks per cylinder, s sectors per track and
a sector length sl. A logical file dl with fixed record length rl is stored
continuously on this disk starting at location (cL,tL,sL), when CL, tL and SL are
the cylinder, track and sector numbers, respectively. Derive the formula to
calculate the disk address (i.e. cylinder, track and sector) of a logical record
n assuming that rl = sl.

26. Consider the following database relations containing the attributes


Book – id
Subject – Category – of – book
Name – of – Author
Nationality – of – Author
With book – id as the primary key.
(a) What is the highest normal form satisfied by this relation?
(b) Suppose the attributes Book – title and Author – address are added to the
relation, and the primary key is changed to {Name – of – Author, Book –
title}, what will be the highest normal form satisfied by the relation?

27. Consider the following relational database schemes:


COURSES (Cno.name)
PRE-REQ(Cno, pre-Cno)
COMPLETED (student – no, Cno)
COURSES gives the number and name of all the available courses.
PRE-REQ gives the information about which courses are pre-requisites for a given
course.
COMPLETED indicates what courses have been completed by students.
Express the following using relational algebra:
List all the courses for which a student with student-no 2310 has completed all
the pre-requisites.
SECTION - A
1. This question consists of TWENTY-FIVE multiple questions of ONE mark each. For
each question, four possible alternatives (A, B, C and D) are given, out of which
ONLY ONE is correct. Indicate the correct answer in the boxes corresponding to
the questions only on the FIRST sheet of the answer book.

1.1 Suppose that the expectation of a random variable X is 5. Which of the following
statements is true?
(a) There is a sample point at which X has the value 5.
(b) There is a sample point at which X has value greater than 5.
(c) There is a sample point at which X has a value greater than or equal to 5.
(d) None of the above

1.2 The number of binary relations on a set with n elements is:


(a) n2 (b) 2n
2
(c) 2n (d) None of the above

1.3 The number of binary strings of n zeroes and k ones that no two ones are
adjacent is
n −1 n
(a) Ck (b) Ck
n
(c) C k +1 (d) None of the above

1.4 Consider the regular expression (0 + 1) (0 + 1)…. N times. The minimum state
finite automation that recognizes the language represented by this regular
expression contains
(a) n states (b) n + 1 states
(c) n + 2 states (d) None of the above

1.5 Context-free languages are closed under:


(a) Union, intersection (b) Union, Kleene closure
(c) Intersection, complement (d) Complement, Kleene closure

1.6 Let LD be the set of all languages accepted by a PDA by final state and LE the set
of all languages accepted by empty stack. Which of the following is true?
(a) LD = LE (b) LD ⊃ LE
(c) LE = LD (d) None of the above

1.7 Which of the following expressions is not equivalent to x ?


(a) x NAND X (b) x NOR x (c) x NAND 1 (d) x NOR 1

Think GATE Think GATE Forum


1.8 Which of the following functions implements the Karnaugh map shown below?

CD
00 00 11 10
AB
00 0 0 1 0
01 X X 1 X
11 0 1 1 0
10 0 1 1 0

(a) AB + CD (b) D(C +A)

(c) AD + AB ( )
(d) ( C + D ) C + D + ( A + B )

1.9 Listed below are some operating system abstractions (in the left column) and the
hardware components (in the right column)?

(A) Thread 1. Interrupt


(B) Virtual address space 2. Memory
(C) File system 3. CPU
(D) Signal 4. Disk

(a) (A) – 2 (B) – 4 (C) – 3 (D) - 1 (b) (A) – 1 (B) – 2 (C) – 3 (D) – 4
(c) (A) – 3 (B) – 2 (C) – 4 (D) - 1 (d) (A) – 4 (B) – 1 (C) – 2 (D) – 3

1.10 Which of the following disk scheduling strategies is likely to give the best through
put?
(a) Farthest cylinder next (b) Nearest cylinder next
(c) First come first served (d) Elevator algorithm

1.11 System calls are usually invoked by using


(a) a software interrupt (b) polling
(c) an indirect jump (d) a privileged instruction

1.12 A sorting technique is called stable if


(a) it takes O (nlog n) time
(b) it maintains the relative order of occurrence of non-distinct elements
(c) it uses divide and conquer paradigm
(d) it takes O(n) space
1.13 Suppose we want to arrange the n numbers stored in any array such that all
negative values occur before all positive ones. Minimum number of exchanges
required in the worst case is
(a) n - 1 (b) n
(c) n + 1 (d) None of the above

1.14 If one uses straight two-way merge sort algorithm to sort the following elements
in ascending order:
20, 47, 15, 8, 9, 4, 40, 30, 12, 17
then the order of these elements after second pass of the algorithm is:
(a) 8, 9, 15, 20, 47, 4, 12, 17, 30, 40
(b) 8, 15, 20, 47, 4, 9, 30, 40, 12, 17
(c) 15, 20, 47, 4, 8, 9, 12, 30, 40, 17
(d) 4, 8, 9, 15, 20, 47, 12, 17, 30, 40

1.15 The number of articulation points of the following graph is

1
5

2 3 7

6
4

(a) 0 (b) 1 (c) 2 (d) 3

1.16 If n is a power of 2, then the minimum number of multiplications needed to


compute a* is
(a) log2n (b) n (c) n -1 (d) n

1.17 Which of the following is the most powerful parsing method?


(a) LL (1) (b) Canonical LR (c) SLR (d) LALR

1.18 Consider the join of a relation R with a relation S. If R has m tuples and S has n
tuples then the maximum and minimum sizes of the join respectively are
(a) m + n and 0 (b) mn and 0
(c) m + n and |m – n| (d) mn and m + n
1.19. The relational algebra expression equivalent to the following tuple calculus
expression:

{t t ∈ r ∧ (t  A = 10 ∧ t B = 20)} is


(a) σ ( A =10 ∨ B = 20) ( r ) (b) σ ( A =10 ) ( r ) ∪ σ ( B = 20 ) ( r )

(c) σ ( A =10 ) ( r ) ∩ σ ( B = 20 ) ( r ) (d) σ ( A =10 ) ( r ) − σ ( B = 20) ( r )

1.20. Booth’s coding in 8 bits for the decimal number –57 is


(a) 0 – 100 + 1000 (b) 0 – 100 + 100 -1
(c) 0 – 1 + 100 – 10 + 1 (d) 0 0 – 10 + 100 - 1

1.21. The maximum gate delay for any output to appear in an array multiplier for
multiplying two n bit number is

(a) O n2 ( ) (b) O(n) (c) O(log n) (d) O(1)

1.22. The main memory of a computer has 2 cm blocks while the cache has 2 c blocks.
If the cache uses the set associative mapping scheme with 2 blocks per set, then
block k of the main memory maps to the set
(a) (k mod m) of the cache (b) (k mod c) of the cache
(c) (k mod 2c) of the cache (d) (k mod 2 cm) of the cache

1.23. The Newton-Raphson method is to be used to find the root of the equation f(x)=0
where xo is the initial approximation and f ′ is the derivative of f. The method
converges
(a) always (b) only if f is a polynomial
(c) only if f ( xo ) < 0 (d) None of the above

1.24. Let R = (a, b, c, d, e, f) be a relation scheme with the following dependencies


c  f, e  a, ec  d. a  b. Which of the following is a key for R?
(a) CD (b) EC (c) AE (d) AC

1.25 Which of the following is correct?


(a) B-trees are for storing data on disk and B+ trees are for main memory.
(b) Range queries are faster on B* trees.
(c) B-trees are for primary indexes and B* trees are for secondary indexes.
(d) The height of a B* tree is independent of the number of records.
2. This question consists of TWENTY-FIVE sub-questions, of TWO marks each. For
each of these sub-questions, four possible alternatives (A, B, C and D) are given,
out of which ONLY ONE is correct. Indicate the correct answers in the boxes
corresponding to the questions only on the SECOND sheet of the answer book.
1
2.1 Consider two events E1 and E2 such that probability of E1, Pr [E1]= , probability
2
1 1
of E2, Pr E21  =
, and probability of E1 and E2, Pr[E1 and E2]= . Which of the
3 5
following statements is/are true?
2
(a) Pr E1 or E2  is
3
(b) Events E1 and E2 are independent
(c) Events E1 and E2 are not independent
E  4
(d) Pr  1  =
 E2  5

2.2. Two girls have picked 10 roses, 15 sunflowers and 15 daffodils. What is the
number of ways they can divide the flowers amongst themselves?
(a) 1638 (b) 2100
(c) 2640 (d) None of the above

2.3. Let L be a set with a relation R which is transitive, anti-symmetric and reflexive
and for any two elements a, b ∈ L let the least upper bound lub (a,b) and the
greatest lower bound glb (a,b) exist. Which of the following is/are true?
(a) L is a poset (b) L is a Boolean algebra
(c) -L1 is context free (d) -L2 is regular

2.4. If L is context free language and L2 is a regular language which of the following
is/are false?
(a) L1 – L2 is not context free (b) L1 ∩ L2 is context free
(c) ~L1 is context free (d) ~L2 is regular

2.5. Given the programming constructs (i) assignment (ii) for loops where the loop
parameter cannot be changed within the loop (iii) if-then-else (iv) forward go to
(v) arbitrary go to (vi) non-recursive procedure call (vii) recursive
procedure/function call (viii) repeat loop, which constructs will you not include in
a programming language such that it should be possible to program the
terminates (i.e., halting) function in the same programming language.
(a) (ii), (iii), (iv)
(b) (v), (vii), (viii)
(c) (vi), (vii), (viii)
(d) (iii), (vii), (viii)
2.6. For the schedule given below, which of the following is correct:
1 Read
2 Read B
3 Write A
4 Read A
5 Write A
6 Write B
7 Read B
8 Write B
(a) This schedule is serialized and can occur in a scheme using 2PL protocol
(b) This schedule is serializable but cannot occur in a scheme using 2PL protocol
(c) This schedule is not serialiable but can occur in a scheme using 2PL protocol
(d) This schedule is not seralisable and cannot occur in a scheme using 2PL
protocol.

2.7. Consider the schema R = (S T U V) and the dependencies S  T, T  U. U  V


and V  S. Let R = (R1 and R2) be a decomposition such that R1 ∩ R2 = ∅ . The
decomposition is
(a) not in 2NF (b) in 2NF but not 3NF
(c) in 3NF but not in 2NF (d) in both 2NF and 3NF

2.8. Consider the circuit shown below. In a certain steady state, the line Y is at ‘1’.
What are the possible values of A, B and c in this state?

A Y

B
C

(a) A = 0, B = 0, C = 1 (b) A = 0, B = 1, C = 1
(c) A = 1, B = 0, C = 1 (d) A = 1, B = 1, C = 1

2.9. Which of the following sets of component(s) is/are sufficient to implement any
arbitrary Boolean function?
(a) XOR gates, NOT gates
(b) 2 to 1 multiplexors
(c) AND gates, XOR gates
(d) Three-input gates that output (A.B) + C for the inputs A. B and C.
2.10. A multi-user, multi-processing operating system cannot be implemented on
hardware that does not support
(a) Address translation
(b) DMA for disk transfer
(c) At least two modes of CPU execution (privileged and non-privileged)
(d) Demand paging

2.11. Which of the following is/are advantage of virtual memory?


(a) Faster access to memory on an average.
(b) Processes can be given protected address spaces.
(c) Linker can assign addresses independent of where the program will be loaded
in physical memory.
(d) Programs larger than the physical memory size can be run.

2.12. Which of the following actions is/are typically not performed by the operating
system when switching context from process A to process B?
(a) Saving current register values and restoring saved register values for process
B.
(b) Changing address translation tables.
(c) Swapping out the memory image of process A to the disk.
(d) Invalidating the translation look-aside buffer.

2.13. Consider the following program in a language that has dynamic scooping:
var x: real;
procedure show:
begin print(x);end;
procedure small;
var x: real;
begin x: = 0.125; show; end;
begin x:=0.25;
show; small
end.
Then the output of the program is:
(a) 0.125 0.125 (b) 0.25 0.25 (c) 0.25 0.125 (d) 0.125 0.25

2.14. The number of tokens in the Fortran statement DO 10 I = 1.25 is


(a) 3 (b) 4
(c) 5 (d) None of the above
2.15. A grammar that is both left and right recursive for a non-terminal, is
(a) Ambiguous (b) Unambiguous
(c) Information is not sufficient to decide whether it is ambiguous or
unambiguous
(d) None of the above

2.16. The number of full and half-adders required to add 16-bit numbers is
(a) 8 half-adders, 8 full-adders (b) 1 half-adder, 15 full-adders
(c) 16 half-adders, 0 full-adders (d) 4 half-adders, 12 full-adders

2.17. Zero has two representations in


(a) Sign magnitude (b) 1’s complement
(c) 2’s complement (d) None of the above

2.18. Raid configurations of the disks are used to provide


(a) Fault-tolerance (b) High speed
(c) high data density (d) None of the above

2.19. Arrange the following configuration for CPU in decreasing order of operating
speeds: Hard wired control, vertical microprogramming, horizontal
microprogramming.
(a) Hard wired control, vertical micro-programming, horizontal micro-
programming.
(b) Hard wired control, horizontal micro-programming, vertical micro-
programming.
(c) horizontal micro-programming, vertical micro-programming, Hard wired
control.
(d) vertical micro-programming, horizontal micro-programming, hard wired
control.

2.20. The minimum number of record movements required to merge five files A (with
10 records), B (with 20 records), C (with 15 records), D (with 5 records) and E
(with 25 records) is:
(a) 165 (b) 90 (c) 75 (d) 65

2.21. If T1 = O(1), give the correct matching for the following pairs:

(M) Tn = Tn −1 + n (U) Tn = O ( n )
(N) Tn = Tn + n (V) Tn = O ( n log n )
2
(O) Tn = Tn + n log n
2
(W) Tn = O n2 ( )
(P) Tn = Tn −1 + log n (X) Tn = O (log n )
2
(a) M – W N – V O – U P - X (b) M – W N – U O – X P - V
(c) M – V N – W O – X P - U (d) M – W N – U O – V P - X

2.22. The main differences(s) between a CSIC and A RISC processor is/are that a RISC
processor typically
(a) has fewer instructions (b) has fewer addressing modes
(c) has more registers
(d) is easier to implement using hard-wired control logic

2.23. A certain processor supports only the immediate and the direct addressing
modes. Which of the following programming language features cannot be
implemented on this processor?
(a) Pointers (b) Arrays
(c) Records
(d) Recursive procedures with local variable

2.24. Consider the following C function definition


int Trial (int a, int b, int c)
{
if ((a > = b) && (c < b) return b;
else if (a > = b) return Trial (a,c,b);
else return Trial (b,a,c);
}
The function Trial:
(a) Finds the maximum of a, b, and c (b) Finds the minimum of a, b and c
(c) Finds the middle number of a, b, c (d) None of the above

2.25. Which of the following is/are correct?


(a) An SQL query automatically eliminates duplicates
(b) An SQL query will not work if there are no indexes on the relations
(c) SQL permits attribute names to be repeated in the same relation
(d) None of the above

SECTION – B
This section consists of TWENTY questions of FIVE marks each. Attempt ANY FIFTEEN
questions. If more number of questions are attempted, score off the answer not to be
evaluated, else only the first fifteen unscored answers will be considered.
3. (a) Mr. X claims the following:
If a relation R is both symmetric and transitive, then R is reflexive. For this,
Mr. X offers the following proof:
“From xRy, using symmetry we get yRx. Now because R is transitive xRy and
yRx together imply xRx.
Therefore, R is reflexive”.
(b) Give an example of a relation R which is symmetric and transitive but not
reflexive.

4. Let G be a finite group and H be a subgroup of G. For a ∈ G, define aH={ah1


h∈H).
(a) Show that |aH| = |H|
(b) Show that for every pair of elements a,b ∈G, either aH = bH or aH and bH
are disjoint.
(c) Use the above to argue that the order of H must divide the order of G.

5. Let G be a connected, undirected graph. A cut in G is a set of edges whose


removal results in G being broken into two or more components, which are not
connected with each other. The size of a cut is called its cardinality. A min-cut of
G is a cut in G of minimum cardinality. Consider the following graph.
A E

D
B

C F

(a) Which of the following sets of edges is a cut?


(i) {(A, B), (E, F), (B, D), (A, E), (A, D)]
(ii) {(B, D), (C, F), (A,B)}
(b) What is the cardinality of min-cut in this graph?
(c) Prove that if a connected undirected graph G with n vertices has a min-cut of
cardinality k, then G has at least (nk/2) edges.

6. (a) Given that A is regular and (A ∪B) is regular, does it follow that B is
necessarily regular ? Justify your answer.
(b) Given two finite automata M1, M2, outline an algorithm to decide if L(M1) ⊆
L(M2). (note: strict subset)

7. Show that the language L{xcx 1 x ∈ {0,1}* and c is a terminal symbol} is not
context free c is not 0 or 1.

8. Let Abe an n × n matrix such that the elements in each row and each column are
arranged in ascending order. Draw a decision tree, which finds 1st, 2nd and 3rd
smallest elements in minimum number of comparisons.
9. Let synthesized attribute val give the value of the binary number generated by S
in the following grammar. For example, on input 101, 101, S.val = 5.625.
S  LL\L
L  LB\B
B  0\1
Write S-attributed values corresponding to each of the productions to find S.val.

10. Suppose we have a function HALTS which when applied to any arbitrary function f
and its arguments will say TRUE if function f terminates for those arguments and
FALSE otherwise. Example: Given the following function definition.
FACTORIAL (N) = IF (N = 0) THEN 1 ELSE N*FACTORIAL (N-1)
Then HALTS (FACTORIAL 4) = TRUE and HALTS (FACTORIAL –5)=FALSE
Let us define the function. FUNNY (f) = IF HALTS (f f) THEN not (f f) ELSE TRUE
(a) Show that FUNNY terminates for all functions f.
(b) use (a) to prove (by contradiction) that it is not possible to have a function
like HALTS which for arbitrary functions and inputs says whether it will
terminate on that input or not.

11. (a) Consider the following algorithms. Assume, procedure A and procedure B
take ( ) (1) and ( ) (1/n) unit of time respectively. Derive the time
complexity of the algorithm in ( )-notation.
algorithm what (n)
begin
if n = 1 then call A
else begin
what (n –1);
call B(n)
end
end.
(b) Write a constant time algorithm to insert a node with data D just before the
node with address p of a singly linked list.

12. (a) In binary tree, a full node is defined to be a node with 2 children. Use
induction on the height of the binary tree to prove that the number of full
nodes plus one is equal to the number of leaves.
(b) Draw the min-heap that results from insertion of the following elements in
order into an initially empty min-heap: 7, 6, 5, 4, 3, 2, 1. Show the result
after the deletion of the root of this heap.
13. An instruction pipeline consists of 4 stages – Fetch (F), Decode field (D). Execute
(E) and Result Write (W). The 5 instructions in a certain instruction sequence
need these stages for the different number of clock cycles as shown by the table
below.

No. of clock cycles needed for


Instruction F D E W
1 1 2 1 1
2 1 2 2 1
3 2 1 3 2
4 1 3 2 1
5 1 2 1 2
Find the number of clock cycles needed to perform the 5 instructions.

14. (a) Show that the formual [(~p∨q) ⇒ (q ⇒ P)] is not a tautology.
(b) Let A be a tautology and B be any other formula. Prove that (A∨B) is a
tautology.

15. What will be the output of the following program assuming that parameter
passing is
(i) call by value
(ii) call by reference
(iii) call by copy restore
procedure P{x,y,z};
begin y: y + 1; z: x + x end;
begin
a: = : b : =3;
P(a+b,a,a);
Print (a)
end.

16. Consider the following Pascal program skeleton:


program sort (…);
var a,x,….;
procedure readarray;
vari i, …..;
begin
…:=…a…
end;
procedure exchange (…);
begin
…:=…a…;
…:=…x…;
end;
procedure qsort(….);
var k,v, ….;
function partition (…)…;
var i,j,…;
begin
…:=…a…;
…:=…v…;
end;
begin
:
end:
begin
:
end.
Assume that given points in time during program execution, following procedures
are active: sort, qsort (1,9), qsort (1,3), partition (1,3), exchange (1,3)
Show snapshots of the runtime stack with access links after each of the
activations.

17. Consider the following program fragment in the assembly language of a certain
hypothetical processor. The processor has three general purpose registers R1, R2
and R3. The meanings of the instructions are shown by comments (starting with
;) after the instructions.

X: CMP R1,0 ; Compare R1 and 0, set flags appropriately in status


register
JZZ ; Jump if zero to target Z.
MOV R2, R1 ; Copy contents of R1 to R2
SHR R1 ; Shift right R1 by 1 bit
SHL R1 ; Shift left R1 by 1 bit
CMP R2, R1 ; Compare R2 and R1 and set flag in status register
JZY ; Jump if zero to target Y
INC R3 ; Increment R3 by 1;
Y: SHR R1 ; Shift right R1 by 1 bit
JMP X ; Jump to target X
Z:….
(a) Initially R1, R2 and R3 contain the values 5,0 and 0 respectively, what are
the final values of R1 and R3 when control reaches Z?
(b) In general, if R1, R2 and R3 initially contain the values n, 0, and 0
respectively. What is the final value of R3 when control reaches Z?

18. Design a 2K × 8 (2048 locations, each bit wide) memory system mapped at
addresses (1000 )16 to (17FF )16 for the 8085 processor using four 1K × 4 memory
chips. Each of these chips has the following signal pins:
(i) CS (Chip select, data lines are in high impedance state when it is 1)

(ii) RD (0 for read operation)


(iii) WR (0 for write operation)
(iv) A0, A1, …A9 (input address lines. A0 is the lest significant)
(v) D0, D1, D2, D3 (bi-directional data lines. D0 is the least significant)

19. A certain computer system has the segmented paging architecture for virtual
memory. The memory is byte addressable. Both virtual and physical address
spaces contain 216 bytes each. The virtual address space is divi8ded into 8 non-
overlapping equal size segments. The memory management unit (MMU) has a
hardware segment table, each entry of which contains the physical address of the
page table for the segment. Page table are stored in the main memory and
consists of 2 byte page table entries.
(a) What is the minimum page size in bytes so that the page table for a segment
requires at most one page to store it? Assume that the page size can only be
a power of 2
(b) Now suppose that the pages size is 512 bytes. It is proposed to provide a
TLB (Translation look-aside buffer) for speeding up address translation. The
proposed TLB will be capable of storing page table entries for 16 recently
referenced virtual pages, in a fast cache that will use the direct mapping
scheme. What is the number o tag bits that will need to be associated with
each cache entry
(c) Assume that each page table entry contains (besides other information) 1
valid bit, 3 bits for page protection and 1 dirty bit. How many bits are
available in page table entry for storing the aging information for the page?
Assume that the page size is 512 bytes.

20. (a) A certain processor provides a ‘test and set’ instruction that is used as
follows.
TSET register.flag
This instruction atomically copies flag to register and sets flag to 1. Give
pseudopodia for implementing the entry and exit code to a critical region
using this instruction.
(b) Consider the following solution to the producer-consumer problem suing a
buffer of size 1. Assume that the initial value of account is 0. Also assume
that the testing of count and assignment to count are atomic operations.
Producer
Repeat
Produce an item;
if count = 1 then sleep;
place item in buffer.
Count =1:
Wakeup (Consumer);
Forever
Consumer:
Repeat
if count = 0 then sleep;
Remove item from buffer;
Count = 0
Wakeup (producer);
Consumer item;
Forever:
Show that in this solution it is possible that both the processes are sleeping at
the same time.

21. Consider a B-tree with degree m. that is, the number of children, c, of any
internal node (except the root) is such that m ≤ c ≤ 2m –1. derive the maximum
and minimum number of records in the leaf nodes for such a B-tree with height
h.h≥1. (Assume that the root of a tree is at height 0).

22. Consider the set of relations


EMP (Employee-no. Dept-no, Employee-name, Salary)
DEPT (Dept-no. Dept-name, Location)
Write an SQL query to:
(a) Find all employee names who work in departments located at ‘Calcutta’ and
whose salary is greater than Rs.50,000.
(b) Calculate, for each department number, the number of employees with a
salary greater than Rs.1,00,000.
SECTION - A
1. This question consists of TWENTY-THREE multiple questions of ONE mark each.
For each question (1.1 – 1.23), four possible alternatives (A, B, C and D) are
given, out of which ONLY ONE is correct. Indicate the correct answer in the boxes
corresponding to the questions only on the FIRST sheet of the answer book.

1.1 The minimum number of cards to be dealt from an arbitrarily shuffled deck of 52
cards to guarantee that three cards are from some same suit is
(a) 3 (b) 8 (c) 9 (d) 12

1.2 An n × n array v is defined as follows:


ν i, j  = i − j for all i, j, i ≤ i ≤ n,1 ≤ j ≤ n
The sum of the elements of the array v is

(a) 0 (b) n -1 (c) n2 − 3n + 2 (d) n2


( n + 1)
2

1.3 The determinant of the matrix


2 0 0 0
 
8 1 7 2
is:
2 0 2 0
 
9 0 6 1 
(a) 4 (b) 0 (c) 15 (d) 20

1.4 Let S and T be language over Σ={a,b} represented by the regular expressions
(a+b*)* and (a+b)*, respectively. Which of the following is true?
(a) S ⊂ T (b) T ⊂ S (c) S = T (d) S ∩T = φ

1.5 Let L denotes the language generated by the grammar S  0S0/00.


Which of the following is true?
(a) L = 0+ (b) L is regular but not 0+
(c) L is context free but not regular (d) L is not context free

1.6 The number 43 in 2’s complement representation is


(a) 01010101 (b) 11010101 (c) 00101011 (d) 10101011

1.7 To put the 8085 microprocessor in the wait state


(a) lower the HOLD input (b) lower the READY input
(c) raise the HOLD input (d) raise the READY input
1.8 Comparing the time T1 taken for a single instruction on a pipelined CPU with time
T2 taken on a non-pipelined but identical CPU, we can say that
(a) T1 ≤ T2 (b) T1 ≥ T2 (c) T1 < T2
(d) T1 is T2 plus the time taken for one instruction fetch cycle

1.9 The 8085 microprocessor responds to the present of an interrupt


(a) as soon as the TRAP pin becomes ‘high’
(b) by checking the TRAP pin for ‘high’ status at the end of each instruction each
(c) by checking the TRAP pin for ‘high’ status at the end of the execution of each
instruction.
(d) by checking the TRAP pin for ‘high’ status at regular intervals.

1.10 The most appropriate matching for the following pairs

X: Indirect addressing 1: Loops


Y: Immediate addressing 2: Pointers
Z: Auto decrement addressing 3. Constants
is
(a) X – 3 Y – 2 Z - 1 (b) X – 1 Y – 3 Z - 2
(c) X – 2 Y – 3 Z - 1 (d) X – 3 Y – 1 Z - 2

1.11 The following C declarations


struct node{
int i:
float j;
};
struct node *s[10];
define s to be
(a) An array, each element of which is a pointer to a structure of type node
(b) A structure of 2 fields, each field being a pointer to an array of 10 elements
(c) A structure of 3 fields: an integer, a float, and an array of 10 elements
(d) An array, each element of which is a structure of type node

1.12 The most appropriate matching for the following pairs

X: m=malloc(5); m= NULL; 1: using dangling pointers


Y: free(n); n->value=5; 2: using uninitialized pointers
Z: char *p; *p=’a’; 3. lost memory
is:
(a) X – 1 Y – 3 Z - 2 (b) X – 2 Y – 1 Z - 3
(c) X – 3 Y – 2 Z - 1 (d) X – 3 Y – 1 Z - 2

1.13 The most appropriate matching for the following pairs

X: depth first search 1: heap


Y: breadth-first search 2: queue
Z: sorting 3: stack
is:
(a) X – 1 Y – 2 Z - 3 (b) X – 3 Y – 1 Z - 2
(c) X – 3 Y – 2 Z - 1 (d) X – 2 Y – 3 Z - 1

1.14 Consider the following nested representation of binary trees: (X Y Z) indicates Y


and Z are the left and right sub stress, respectively, of node X. Note that Y and Z
may be NULL, or further nested. Which of the following represents a valid binary
tree?
(a) (1 2 (4 5 6 7)) (b) (1 (2 3 4) 5 6) 7)
(c) (1 (2 3 4)(5 6 7)) (d) (1 (2 3 NULL) (4 5))

1.15 Let s be a sorted array of n integers. Let t(n) denote the time taken for the most
efficient algorithm to determined if there are two elements with sum less than
1000 in s. which of the following statements is true?
(a) t (n) is 0(1) (b) n ≤ t(n) ≤ n log2 n
n n
(c) n log2 n ≤ t(n) <   (d) t(n) =  
2 2

1.16 Aliasing in the context of programming languages refers to


(a) multiple variables having the same memory location
(b) multiple variables having the same value
(c) multiple variables having the same identifier
(d) multiple uses of the same variable

1.17 Consider the following C declaration


struct {
short s [5]
union {
float y;
long z;
} u;
}t;
Assume that objects of the type short, float and long occupy 2 bytes, 4 bytes and
8 bytes, respectively. The memory requirement for variable t, ignoring alignment
considerations, is
(a) 22 bytes (b) 14 bytes (c) 18 bytes (d) 10 bytes

1.18 The number of tokens in the following C statement


printf(“i=%d, &i=%x”,i,&i);
is
(a) 3 (b) 26 (c) 10 (d) 21

1.19. Which of the following derivations does a top-down parser use while parsing an
input string? The input is assumed to be scanned in left to right order.
(a) Leftmost derivation
(b) Leftmost derivation traced out in reverse
(c) Rightmost derivation
(d) Rightmost derivation traced out in reverse

1.20. Which of the following need not necessarily be saved on a context switch
between processes?
(a) General purpose registers (b) Translation look-aside buffer
(c) Program counter (d) All of the above

1.21. Let m[0]…m[4] be mutexes (binary semaphores) and P[0] …. P[4] be processes.
Suppose each process P[i] executes the following:
wait (m[i];wait (m[(i+1) mode 4]);
………
release (m[i]); release (m[(i+1)mod 4]);
This could cause
(a) Thrashing (b) Deadlock
(c) Starvation, but not deadlock (d) None of the above

1.22. B+ -trees are preferred to binary trees in databases because


(a) Disk capacities are greater than memory capacities
(b) Disk access is much slower than memory access
(c) Disk data transfer rates are much less than memory data transfer rates
(d) Disks are more reliable than memory

1.23. Given the relations


employee (name, salary, deptno), and
department (deptno, deptname, address)
Which of the following queries cannot be expressed using the basic relational
algebra operations (σ,π,×, ,∪,∩,−)?

(a) Department address of every employee


(b) Employees whose name is the same as their department name
(c) The sum of all employees’ salaries
(d) All employees of a given department

2. This question consists of TWENTY-SIX multiple questions of TWO marks each.


For each question (2.1 – 2.26), four possible alternatives (A, B, C and D) are
given, out of which ONLY ONE is correct. Indicate the correct answer in the boxes
corresponding to the questions only on the SECOND sheet of the answer book.

2.1 X,Y and Z are closed intervals of unit length on the real line. The overlap of X and
Y is half a unit. The overlap of Y and Z is also half a unit. Let the overlap of X and
Z be k units. Which of the following is true?
(a) k must be 1 (b) k must be 0
(c) k can take any value between 0 and 1 (d) None of the above

2.2. E1 and E2 are events in a probability space satisfying the following constraints:
• Pr(E1) = Pr(E2)
• Pr(E1 ∪ E2) = 1
• E1 and E2 are independent
The value of Pr(E1), the probability of the event E1, is
1 1
(a) 0 (b) (c) (d) 1
4 2

100
2.3. Let S = ∑ i log
i =3
2l
and

100
and T= ∫
2
x log2 xdx

Which of the following statements is true?


(a) S > T (b) S = T
(c) S < T and 2S > T (d) 2S ≤ T

2.4. A polynomial p(x) satisfies the following:


p(1) = p(3) = p(5) = 1
p(2) = p(4) = -1
The minimum degree of such a polynomial is
(a) 1 (b) 2 (c) 3 (d) 4
2.5. A relation R is defined on the set of integers as x Ry iff (x+y) is even. Which of
the following statements is true?
(a) R is not an equivalence relation
(b) R is an equivalence relation having 1 equivalence class
(c) R is an equivalence relation having 2 equivalence classes
(d) R is an equivalence relation having 3 equivalence classes

2.6. Let P(S) denotes the powerset of set S. Which of the following is always true?
(a) P(P(S))=P(S) (b) P(S) ∩P(P(S)) = {φ}
(c) P(S) ∩S = P(S) (d) S ∉P(S)

2.7. Let a, b, c, d be propositions. Assume that the equivalence a ↔ (b V-b) and b ↔


c hold. Then the truth-value of the formula ( a ∧ b ) → ( a ∧ c ) ∨ d is always

(a) True (b) False


(c) Same as the truth-value of b (d) Same as the truth-value of d

2.8. What can be said about a regular language L over {a} whose minimal finite state
automation has two states?
(a) L must be {an|n is odd} (b) L must be {an|n is even}
(c) L must be {an|≥0}
(d) Either L must be {an|n is odd}, or L must be {an| n is even}

2.9. Consider the following decision problems:


(P1) Does a given finite state machine accept a given string
(P2) Does a given context free grammar generate an infinite number of stings
Which of the following statements is true?
(a) Both (P1) and (P2) are decidable
(b) Neither (P1) nor (P2) are decidable
(c) Only (P1) is decidable (d) Only (P2) is decidable

2.10. The simultaneous equations on the Boolean variables x, y, z and w,


x +y + z =1
xy = 0
xz + w = 1
xy + z w=0
have the following solution for x, y, z and w, respectively:
(a) 0 1 0 0 (b) 1 1 0 1 (c) 1 0 1 1 (d) 1 0 0 0
2.11. Which functions does NOT implement the Karnaugh map given below?

wz  00 01 11 10
xy↓
00 0 × 0 0
01 0 × 1 1
11 1 1 1 1
10 0 × 0 0

(a) (w + x ) y (b) xy + yw

(c) (w + x ) (w + y ) ( x + y ) (d) None of the above

P Q 1
2.12. The following arrangement of master-slave flip flops

J K D
1

Clock

has the initial state of P, Q as 0, 1 (respectively). After the clock cycles the
output state P, Q is (respectively),
(a) 1, 0 (b) 1, 1 (c) 0, 0 (d) 0, 1

2.13. A graphics card has on board memory of 1 MB. Which of the following modes can
the card not support?
(a) 1600 × 400 resolution with 256 colours on a 17 inch monitor
(b) 1600 × 400 resolution with 16 million colours on a 14 inch monitor
(c) 800 × 400 resolution with 16 million colours on a 17 inch monitor
(d) 800 × 800 resolution with 256 colours on a 14 inch monitor

2.14. Consider the values of A = 2.0 × 1030, B = -2.0 × 1030, C = 1.0, and the sequence
X: = A + B Y:= A + c
X: = X + C Y:= Y + B
Executed on a computer where floating point numbers are represented with 32
bits. The values for X and Y will be
(a) X = 1.0, Y = 1.0 (b) X = 1.0, Y = 0.0
(c) X = 0.0, Y = 1.0 (d) X = 0.0, Y = 0.0
2.15. Suppose you are given an array s[1…n] and a procedure reverse (s,i,j) which
reverses the order of elements in a between positions i and j (both inclusive).
What does the following sequence do, where 1 ≤ k ≤n:
reverse (s, 1, k);
reverse (s, k + 1, n);
reverse (s, 1, n);
(a) Rotates s left by k positions (b) Leaves s unchanged
(c) Reverses all elements of s (d) None of the above

2.16. Let LASTPOST, LASTIN and LASTPRE denote the last vertex visited in a
postorder, inorder and preorder traversal. Respectively, of a complete binary
tree. Which of the following is always tree?
(a) LASTIN = LASTPOST (b) LASTIN = LASTPRE
(c) LASTPRE = LASTPOST (d) None of the above

2.17. Consider the following functions

f ( n ) = 3n n

g ( n) = 2 n log2 n

h ( n) = n!

Which of the following is true?


(a) h(n) is O (f(n)) (b) h(n) is O (g(n))
(c) g(n) is not O (f(n)) (d) f(n) is O(g(n))

2.18. Let G be an undirected connected graph with distinct edge weight. Let emax be the
edge with maximum weight and emin the edge with minimum weight. Which of
the following statements is false?
(a) Every minimum spanning tree of G must contain emin
(b) If emax is in a minimum spanning tree, then its removal must disconnect G
(c) No minimum spanning tree contains emax
(d) G has a unique minimum spanning tree

2.19. Lt G be an undirected graph. Consider a depth-first traversal of G, and let T be


the resulting depth-first search tree. Let u be a vertex in G and let ν be the first
new (unvisited) vertex visited after visiting u in the traversal. Which of the
following statements is always true?
(a) {u,v} must be an edge in G, and u is a descendant of v in T
(b) {u,v} must be an edge in G, and v is a descendant of u in T
(c) If {u,v} is not an edge in G then u is a leaf in T
(d) If {u,v} is not an edge in G then u and v must have the same parent in T
2.20. The value of j at the end of the execution of the following C program
int incr (int i)
{
static int count = 0;
count = count + i;
return (count);
}
main () {
int i,j;
for (i = 0; i <=4; i++)
j = incr(i);
}
is
(a) 10 (b) 4 (c) 6 (d) 7

2.21. Given the following expression grammar:


EE*F|F+E|F
F  F - | id
Which of the following is true?
(a) * has higher precedence than + (b) - has higher precedence than *
(c) + and – have same precedence (d) + has higher precedence than *

2.22. Suppose the time to service a page fault is on the average 10 milliseconds, while
a memory access takes 1 microsecond. Then a 99.99% hit ratio results in
average memory access time of
(a) 1.9999 milliseconds (b) 1 millisecond
(c) 9.999 microseconds (d) 1.9999 microseconds

2.23. Which of the following is NOT a valid deadlock prevention scheme?


(a) Release all resources before requesting a new resource
(b) Number the resources uniquely and never request a lower numbered
resource than the last one requested.
(c) Never request a resource after releasing any resource
(d) Request and all required resources be allocated before execution.
2.24. Given the following relation instance
X Y Z
1 4 2
1 5 3
1 6 3
3 2 2
Which of the following functional dependencies are satisfied by the instance?
(a) XY  Z and Z  Y (b) YZ  X and Y  Z
(c) YZ  X and X  Z (d) XZ  Y and Y  X

2.25. Given relations r(w,x) and s(y,z), the result of


select distinct w,x
from r, s
is guaranteed to be same as r, provided
(a) r has no duplicates and s is non-empty
(b) r and s have no duplicates
(c) s has no duplicates and r is non-empty
(d) r and s have the same number of tuples

2.26 In SQL, relations can contain null values, and comparisons with null values are
treated as unknown. Suppose all comparisons with a null value are treated as
false. Which of the following pairs is not equivalent?
(a) x = 5 not (not (x = 5)
(b) x = 5 x > 4 and x < 6, where x is an integer
(c) x ≠ 5 not (x = 5)
(d) None of the above

SECTION – B
This section consists of TWENTY questions of FIVE marks each. Attempt ANY FIFTEEN
questions. If more number of questions are attempted, score off the answer not to be
evaluated, else only the first fifteen unscored answers will be considered.

3. Consider the following sequence: s1 : s2 =1 and s1 = 1 + mine {si −1 , si − 2 } for i >


n
2. Prove by induction on a n that sn =   .
2 

4. Let S = {0, 1, 2, 3, 4, 5, 6, 7} and ⊗ denote multiplication modulo 8, that is , x


⊗ y = (xy) mod 8
(a) Prove that ({0,1}, ⊗) is not a group.
(b) Write 3 distinct groups (G, ⊗) where G ⊂ S and G has 2 elements.
5. A multiset is an unordered collection of elements where elements may repeat any
number of times. The size of a multiset is the number of elements in it counting
repetitions.
(a) What is the number of multisets of size 4 that can be constructed from n
distinct elements so that at least one element occurs exactly twice?
(b) How many multisets can be consctructed from n distinct elements?

6. Let S be a set of n elements {1, 2, …., n} and G a graph with 2n vertices, each
vertex corresponding to a distinct subset of S. Two vertices are adjacent iff the
symmetric difference of the corresponding sets has exactly 2 elements. Note: The
R  R 
symmetric difference of two sets R1 and R2 is defined as  1  ∪  2 
 R2   R1 
(a) Every vertex in G has the same degree. What is the degree of a vertex in G?
(b) How many connected components does G have?

7. (a) Construct as minimal finite state machine that accepts the language, over
{0,1}, of all strings that contain neither the substring 00 nor the substring
11.
(b) Consider the grammar
S aS Ab
S ε
A bA
A ε
Where S, A are non-terminal symbols with S being the start symbol; a,b are
terminal symbols and ε is the empty string. This grammar generates strings of
the form ai b j for some i, j ≥ 0, where i and j satisfy some condition. What is the
condition on the values of i and j?

8. A pushdown automaton (pda) is given in the following extended notation of finite


state diagrams:
2.s/s
q0 q1

1.s/1.s 1,1.s/s
The nodes denote the states while the edges denote the moves of the pda. The
edge labels are of the form d, s / s ′ where d is the input symbol read and s, s ′ are
the stack contents before and after the move. For example the edge labeled 1,
s/1.s denotes the move from state qo to qo in which the input symbol 1 is read
and pushed to the stack.
(a) Introduce two edges with appropriate labels in the above diagram so that the
resulting pda accepts the language

{ x2 x R
}
x ∈ {0,1} *, x R denotes revese of x , by empty stack.
(b) Describe a non-deterministic pda with three states in the above notation that
{ }
accept the language 0n1m n ≤ m ≤ 2n by empty stack

9. Design a logic circuit to convert a single digit BCD number to the number modulo
six as follows (Do not detect illegal input):
(a) Write the truth table for all bits. Label the input bits I1, I2, …. With I1 as the
least significant bit. Label the output bits R1, R2, …. With R1 as the least
significant bit. Use 1 to signify truth.
(b) Draw one circuit for each output bit using, altogether, two two-input AND
gates, one two-input gate and two NOT gates.

10. Consider the 8085 instruction IN 09H stored as follows:

Memory Address Machine Code

3050 DA
3051 09

And the following incomplete timing diagram for the instruction:


T1 T2 T3 T4 T5 T6 T7 T8 T9 TA

A15-A8 30H 30H C

AD7 - AD0 50H A D B

IO/ M

(a) Write the contents of the boxes, A, B, C and D in hexadecimal in your answer
sheet. Do not draw any pictures.
(b) Write the state of both ALE and RD pins at time units T1, T2, T3 and T4.
(c) How do you generate the signal that tells the peripheral to put the data on
the bus? Answer by completing the following statement in your answer book:
By combining signals …………….

11. Consider the following 8085 program segment, where registers B and C contain
BCD values:
S1: MVI A, 99H
MVI D, 00H
SUB C
ADD B
DAA
S2: JC S3
MOV E, A
MVI A, 99H
SUB E
MOV E, A
JZ S4
MVI D, FFH
JMP S4
S3: INC A
DAA
MOV E, A
S4: …………
(a) For the two pairs (B = 44, C = 25) and (B = 33, C = 46) at S1,
(i) Find the values in register A when control reaches S2.
(ii) Find the values in registers D and E when control reaches S4.
(b) What, in general, is the value of D and E as a function of B and C when
control reaches S4.

12. An instruction pipeline has five stages where each stage takes 2 nanoseconds
and all instructions use all five stages. Branch instructions are not overlapped,
i.e., the instruction after the branch is not fetched till the branch instruction is
completed. Under ideal conditions.
(a) Calculate the average instruction execution time assuming that 20% of all
instruction executed are branch instructions. Ignore the fact that some
branch instructions may be conditional.
(b) If a branch instruction is a conditional branch instruction, the branch need
not be taken. If the branch is not taken, the following instructions can be
overlapped. When 80% of all branch instructions are conditional branch
instructions, and 50% of the conditional branch instructions are such that the
branch is taken, calculate the average instruction execution time.

13. Suppose a stack implementation supports, in addition to PUSH and POP, an


operation REVERSE, which reverses the order of the elements on the stack.
(a) To implement a queue using the above stack implementation, show how to
implement ENQUEUE using a single operation and DEQUEUE using a
sequence of 3 operations.
(b) The following postfix expression, containing single digit operands and
arithmetic operators + and *, is evaluated using a stack.
52*34+52**+
Show the contents of the stack.
(i) After evaluating 5 2 * 3 4 +
(ii) After evaluating 5 2 * 3 4 + 5 2
(iii) At the end of evaluation.
n
14. Consider the line y = x, where n and m are positive integers.
m
(a) If mq – np < 0, then is the point (p,q) above the line, below the line, or on
the line?
(b) Complete the following function, that returns true if the line segment with
n
endpoints (p,q) and (r,s) intersects the line y = x, by writing the line
m
number and the content of each box in your answer book.
1: function clash (m, n, p, q, r, s: integer): Boolean;
2: begin
3: clash = false;
4: if (m*q – n * p) 0 then clash : = true;
5: If (m*s – n * r) 0 then clash : = true;
6: if (m*q – n * p) 0 and (m*s – n * r) 0 then clash : = true;
7: if (m*q – n * p) 0 and (m*s – n * r) 0 then clash : = true;
8: end;

15. Suppose you are given arrays p[1…..N] and q[1…….N] both uninitialized that is,
each location may contain an arbitrary value), and a variable count, initialized to
0. Consider the following procedures set and iset:
set (i) {
count = count + 1;
q [count] = i;
p[i] = count;
}
is_set(i) {
if (p[i] ≤ 0 or p[i] > count)
return false;
if (q[p[i]] ≠i)
return false;
return true;
}
(a) Suppose we make the following sequence of calls:
set (7); set (3); set(9);
After these quence of calls, what is the value of count, and what do q[1],
q[2], q[3], p[7], p[3] and p[9] contain?
(b) Complete the following statement “The first count elements of _______
contain values i such that set ( __________) has been called”.
(c) Show that if set (i) has not been called for some i, then regardless of what
p[i] contains, is_set (i) will return false.
16. A recursive program to compute Fibonacci numbers is shown below. Assume you
are also given an array f[0…..m] with all elements initialized to 0.
fib(n) {
if (n > M) error ();
if ( n==0) return 1;
if (n ==1) return 1;
if ( ) _________________(1)
return ____________(2)
t = fib(n – 1) + fib (n – 2);
__________(3)
return t;
}
(a) Fill in the boxes with expressions/statements to make fib() store and reuse
computed Fibonacci values. Write the box number and the corresponding
contents in your answer book.
(b) What is the time complexity of the resulting program when computing fib(n)?

17. An array contains four occurrences of 0, five occurrences of 1, and three


occurre3nces of 2 in any order. The array is to be sorted using swap operations
(elements that are swapped need to be adjacent).
(a) What is the minimum number of swaps needed to sort such an array in the
worst case?
(b) Give an ordering of elements in the above array so that the minimum
number of swaps needed to sort the array is maximum.

18. Consider the following program is pseudo-Pascal syntax.


program main;
var x: integer;
procedure Q [z:integer);
begin
z: z + x;
writeln(z)
end;
procedure P (y:integer);
var x: integer;
begin
x: y + 2;
Q(x);
writeln(x)
end;
begin
x:=5;
P(x);
Q(x);
writeln(x)
end.

What is the output of the program, when


(a) The parameter passing mechanism is call-by-value and the scope rule is
static scooping?

(b) The parameter passing mechanism is call-by-reference and the scope rule is
dynamic scooping?

19. Consider the syntax directed translation scheme (SDTS) given in the following.
Assume attribute evaluation with bottom-up parsing, i.e., attributes are
evaluated immediately after a reduction.
E  E1 * T {E.val = E1. val * T. val}
E  T {E. val = T. val}
T  F – T1 {T.val = F. val – T1. val}
T  F {T. val = F. val}
F  2 {F. val =2}
F  4 {F. val =4}

(a) Using this SDTS, construct a parse tree for the expression
4–2–4*2
and also compute its E.val.

(b) It is required to compute the total number of reductions performed to parse


a given input. Using synthesized attributes only, modify the SDTS given,
without changing the grammar, to find E.red, the number of reductions
performed while reducing an input to E.

20. (a) Fill in the boxes below to get a solution for the readers-writers problem,
using a single binary semphore, mutex (initialized to 1) and busy waiting.
Write the box numbers (1,2 and 3), and their contents in your answer book.
int R = 0, W = 0;
Reader ( ) {
L1: wait (mutex);
If (W ==0) {
R = R +1;
_______(1)
}
else {
_______(2)
goto L1;
}
…./* do the read */
wait (mutex)
R = R – 1;
signal (mutex);
}
Writer () {
L2: wait(mutex);
If ( ) { ____(3)
signal (mutex);
goto L2;
}
W=1;
signal (mutex);
…./*do the write*/
wait(mutex)
W = 0;
signal (mutex);

(b) Can the above solution lead to starvation of writers?

21. (a) Suppose you are given an empty B+-tree where each node (leaf and
internal) can store up to 5 key values. Suppose values 1,2,….. 10 are
inserted, in order, into the tree, Show the tree pictorially
(i) After 6 insertions, and
(ii) After all 10 insertions
Do NOT show intermediate stages.
(b) Suppose instead of splitting a node when it is full, we try to move a value to
the left sibling. If there is no left sibling, or the left sibling is full, we split the
node. Show the tree after values, 1, 2,….., 9 have been inserted. Assume, as
in (a) that each node can hold up to 5 keys.
(c) In general, suppose a B+-tree node can hold a maximum of m keys, and you
insert a long sequence of keys in increasing order. Then what approximately
is the average number of keys in each leaf level node.
(i) In the normal case, and
(ii) With the insertion as in (b).
22. Consider a bank database with only one relation
transaction (transno, acctno, date, amount)
The amount attribute value is positive for deposits and negative for withdrawals.

(a) Define an SQL view TP containing the information.


(acctno, T1.date, T2.amount)
for every pair of transactions T1, T2 such that T1 and T2 are transaction on
the same account and the date of T2 is ≤ the date of T1.
(b) Using only the above view TP, write a query to find for each account the
minimum balance it ever reached (not including the 0 balance when the
account is created). Assume there is at most one transaction per day on each
account, and each account has had atleast one transaction since it was
created. To simply your query, break it up into 2 steps by defining an
intermediate view V.
SECTION - A
1. This question consists of TWENTY-FIVE sub-questions (1.1 – 1.25) of ONE mark
each. For each of these sub-questions, four possible alternatives, A, B, C and D
are provided. Choose the most appropriate alternative and darken its bubble on
the Objective Response Sheet (ORS) against the corresponding sub-question
number using a soft HB pencil. Do not darken more than one bubble for any
sub-question. Do not use the ORS for any rough work. You may use the answer
book for any rough work, if needed.
1.1 Consider the following statements:
S1: The sum of two singular n × n matrices may be non-singular
S2: The sum of two n × n non-singular matrices may be singular.
Which of the following statements is correct?
(a) S1 and S2 are both true (b) S1 is true, S2 is false
(c) S1 is false, S2 is true (d) S1 and S2 are both false

1.2 Consider the following relations:


R1 (a,b) iff (a+b) is even over the set of integers
R2 (a,b) iff (a+b) is odd over the set of integers
R3 (a,b) iff a.b > 0 over the set of non-zero rational numbers
R4 (a,b) iff |a – b| ≤ 2 over the set of natural numbers
Which of the following statements is correct?
(a) R1 and R2 are equivalence relations, R3 and R4 are not
(b) R1 and R3 are equivalence relations, R2 and R4 are not
(c) R1 and R4 are equivalence relations, R2 and R3 are not
(d) R1, R2, R3 and R4 are all equivalence relations

1.3 Consider two well-formed formulas in prepositional logic


F1: P ⇒ ¬P F2: (P⇒¬P)∨(¬P⇒P)
Which of the following statements is correct?
(a) F1 is satisfiable, F2 is valid (b) F1 unsatisfiable, F2 is satisfiable
(c) F1 is unsatisfiable, F2 is valid (d) F1 and F2 are both satisfiable

1.4 consider the following two statements:

{ }
S1: 02n n ≥ 1 is a regular language

S2: {0 m n m+ n
1 0 }
m ≥ 1 and n ≥ 1 is a regular language

Which of the following statements is correct?


(a) Only S1 is correct (b) Only S2 is correct
(c) Both S1 and S2 are correct (d) None of S1 and S2 is correct
1.5 Which of the following statements s true?
(a) If a language is context free it can always be accepted by a deterministic
push-down automaton
(b) The union of two context free languages is context free
(c) The intersection of two context free languages is context free
(d) The complement of a context free language is context free

1.6 Given an arbitary non-deterministic finite automaton (NFA) with N states, the
maximum number of states in an equivalent minimized DFA is at least
(a) N2 (b) 2N (c) 2N (d) N!

1.7 More than one word are put in one cache block to
(a) exploit the temporal locality of reference in a program
(b) exploit the spatial locality of reference in a program
(c) reduce the miss penalty
(d) none of the above

1.8 Which of the following statements is false?


(a) Virtual memory implements the translation of a program’s address space into
physical memory address space
(b) Virtual memory allows each program to exceed the size of the primary
memory
(c) Virtual memory increases the degree of multiprogramming
(d) Virtual memory reduces the context switching overhead

1.9 A low memory can be connected to 8085 by using


(a) INTER (b) RESET IN (c) HOLD (d) READY

1.10 Suppose a processor does not have any stack pointer register. Which of the
following statements is true?
(a) It cannot have subroutine call instruction
(b) It can have subroutine call instruction, but no nested subroutine calls
(c) Nested subroutine calls are possible, but interrupts are not
(d) All sequences of subroutine calls and also interrupts are possible
1.11 Given the following Karnaugh map, which one of the following represents the
minimal Sum-Of-Products of the map?

wx
yz 00 01 11 10
00 0 X 0 X
01 X 1 X 1
11 0 X 1 0
10 0 1 X 0

(a) xy + y ′z (b) wx ′y ′ + xy + xz (c) w ′x + y ′z + xy (d) xz+y

1.12 A processor needs software interrupt to


(a) test the interrupt system of the processor
(b) implement co-routines
(c) obtain system services which need execution of privileged instructions
(d) return from subroutine

1.13 A CPU has two modes-privileged and non-privileged. In order to change the mode
from privileged to non-privileged
(a) a hardware interrupt is needed
(b) a software interrupt is needed
(c) a privileged instruction (which does not generate an interrupt) is needed
(d) a non-privileged instruction (which does not generate an interrupt is needed

1.14 Randomized quicksort is an extension of quicksort where the pivot is chosen


randomly. What is the worst case complexity of sorting n numbers using
randomized quicksort?
(a) O(n) (b) O(n log n) (c) O(n2) (d) O(n!)

1.15 Consider any array representation of an n element binary heap where the
elements are stored from index 1 to index n of the array. For the element stored
at index i of the array (i≤n), the index of the parent is

(a) i-1 (b) 


i
 (c) 
i
 (d)
( i + 1)
2 2 2
10
1.16 Let f ( n ) = n2 log n and g ( n ) = n (log n ) be two positive functions of n. Which of
the following statements is correct?
(a) f(n) = O(g(n) and g(n) ≠O(f(n)) (b) g(n) = O(f(n) and f(n) ≠O(g(n))
(c) f(n)≠O(g(n)) and g(n) ≠O(f(n)) (d) f(n)=O(g(n)) and g(n) =O(f(n))

1.17 The process of assigning load addresses to the various parts of the program and
adjusting the code and date in the program to reflect the assigned addresses is
called
(a) Assembly (b) Parsing
(c) Relocation (d) Symbol resolution

1.18 Which of the following statements is false?


(a) An unambiguous grammar has same leftmost and rightmost derivation
(b) An LL(1) parser is a top-down parser
(c) LALR is more powerful than SLR
(d) An ambiguous grammar can never be LR(k) for any k

1.19 Consider a set of n tasks with known runtimes r1, r2, …. rn to be run on a
uniprocessor machine. Which of the following processor scheduling algorithms will
result in the maximum throughput?
(a) Round-Robin (b) Shortest-Job-First
(c) Highest-Response-Ratio-Next (d) First-Come-First-Served

1.20 Where does the swap space reside?


(a) RAM (b) Disk
(c) ROM (d) On-chip cache

1.21 Consider a virtual memory system with FIFO page replacement policy. For an
arbitrary page access pattern, increasing the number of page frames in main
memory will
(a) always decrease the number of page faults
(b) always increase the number of page faults
(c) sometimes increase the number of page faults
(d) never affect the number of page faults

1.22 Which of the following requires a device driver?


(a) Register (b) Cache (c) Main memory (d) Disk
1.23 Consider a schema R(A,B,C,D) and functional dependencies A  B and C  D.
Then the decomposition of R into R1 (AB) and R2(CD) is
(a) dependency preserving and lossless join
(b) lossless join but not dependency preserving
(c) dependency preserving but not lossless join
(d) not dependency preserving and not lossless join

1.24 Suppose the adjacency relation of vertices in a graph is represented in a table


Adj (X,Y). Which of the following queries cannot be expressed by a relational
algebra expression of constant length?
(a) List of all vertices adjacent to a given vertex
(b) List all vertices which have self loops
(c) List all vertices which belong to cycles of less than three vertices
(d) List all vertices reachable from a given vertex

1.25 Let r and s be two relations over the relation schemes R and S respectively, and
let A be an attribute in R. then the relational algebra expression σ A = a (r s) is
always equal to
(a) σ A = a (r) (b) r
(c) σ A = a (r) s (d) None of the above

2. This question consists of TWENTY-FIVE sub-questions (2.1 – 2.25) of TWO marks


each. For each of these sub-questions, four possible alternatives, A,B, C and D
are provided. Choose the most appropriate alternative and darken its bubble on
the Objective Response Sheet (ORS) against the corresponding sub-question
number using a soft HB pencil. Do not darken more than one bubble for any
sub-question. Do not use the ORS for any rough work. You may use the answer
book for any rough work, if needed.

2.1 How many 4-digit even numbers have all 4 digits distinct?
(a) 2240 (b) 2296 (c) 2620 (d) 4536

2.2 Consider the following statements:


S1: There exists infinite sets A, B, C such that A∩(B∪C) is finite.
S2: There exists two irrational numbers x and y such that (x+y) is rational.
Which of the following is true about S1 and S2?
(a) Only S1 is correct (b) Only S2 is correct
(c) Both S1 and S2 are correct (d) None of S1 and S2 is correct
2.3 Let f: A  B be a function, and let E and F be subsets of A. Consider the following
statements about images.
S1:f(E∪F)=f(E) ∪f(F)
S2:f(E∩F)=f(E) ∩f(F)
Which of the following is true about S1 and S2?
(a) Only S1 is correct (b) Only S2 is correct
(c) Both S1 and S2 are correct (d) None of S1 and S2 is correct

2.4 Seven (distinct) car accidents occurred in a week. What is the probability that
they all occurred on the same day?
1 1 1 7
(a) (b) (c) (d)
77 76 27 27

2.5 Consider a DFA over ∑ = {a, b} accepting all strings which have number of a’s
divisible by 6 and number of b’s divisible by 8. What is the minimum number of
states that the DFA will have?
(a) 8 (b) 14 (c) 15 (d) 48

2.6 Consider the following languages:

{
L1 = ww w ∈ {a, b} * }
{
L2 = ww R w ∈ {a, b} *, w R is the reverse of w }
{
L3 = 02i i is an integer }
{ 2
L3 = 0i i is an integer }
Which of the languages are regular?
(a) Only L1 and L2 (b) Only L2, L3 and L4
(c) Only L3 and L4 (d) Only L3

2.7 Consider the following problem X.


Given a Turing machine M over the input alphabet Σ, any state q of M
And a word w ∈Σ*, does the computation of M on w visit the state q?
Which of the following statements about X is correct?
(a) X is decidable
(b) X is undecidable but partially decidable
(c) X is undecidable and not even partially decidable
(d) X is not a decision problem
2.8 Consider the following circuit with initial state Q0 = Q1 = 0. The D Flip-flops are
positive edged triggered and have set up times 20 nanosecond and hold times 0.

D1 Y
X D0 Q1

Q0
Clock Clock

C
Consider the following timing diagrams of X and C; the clock period of C≥40
nanosecond. Which one is the correct plot of Y?

(a)

(b)

(c)

(d)

2.9 Which is the most appropriate match for the items in the first column with the
items in the second column

X. Indirect Addressing I. Array implementation


Y. Indexed Addressing II. Writing re-locatable code
Z. Base Register Addressing III. Passing array as parameter

(a) (X, III) (Y, I) (Z, II) (b) (X, II) (Y, III) (Z, I)
(c) (X, III) (Y, II) (Z, I) (d) (X, I) (Y, III) (Z, II)
2.10 The 2’s complement representation of ( −539 )10 is hexadecimal is

(a) ABE (b) DBC (c) DE5 (d) 9E7

2.11 Consider the circuit shown below. The output of a 2:1 Mux is given by the
function ( ac ′ + bc ) .

1 a 2:1 a 2:1
MUX g
MUX f
0 b
c b c

x1 x2

Which of the following is true?


(a) f = x1′ + x2 (b) f = x1′ x2 + x1x2′
(c) f = x1x2 + x1′ x2′ (d) f = x1 + x2′

2.12 Consider the circuit given below with initial state Q0 =1, Q1 = Q2 = 0. The state of
the circuit is given by the value 4Q2 + 2Q1 + Q0

D0 Q0 D1 Q1 D2 Q2

LSB MSB

Clock

Which one of the following is the correct state sequence of the circuit?
(a) 1,3,4,6,7,5,2 (b) 1,2,5,3,7,6,4 (c) 1,2,7,3,5,6,4 (d) 1,6,5,7,2,3,4
2.13 Consider the following data path of a simple non-pilelined CPU. The registers A,
B, A1, A2, MDR, the bus and the ALU are 8-bit wide. SP and MAR are 16-bit
registers. The MUX is of size 8 × (2:1) and the DEMUX is of size 8 × (1:2). Each
memory operation takes 2 CPU clock cycles and uses MAR (Memory Address
Register) and MDR (Memory Date Register). SP can be decremented locally.

A2 A1
B A MUX DEMUX
1:2 1:2

dcr SP MAR MDR

The CPU instruction “push r”, where = A or B, has the specification


M [SP]  r
SP  SP – 1
How many CPU clock cycles are needed to execute the “push r” instruction?
(a) 2 (b) 3 (c) 4 (d) 5

2.14 Consider an undirected unweighted graph G. Let a breadth-first traversal of G be


done starting from a node r. Let d(r,u) and d(r,v) be the lengths of the shortest
paths from r to u and v respectively in G. If u is visited before v during the
breadth-first traversal, which of the following statements is correct?
(a) d(r,u)<d(r,v) (b) d(r,u)>d(r,v)
(c) d(r,u)≤d(r,v) (d) None of the above

2.15 How many undirected graphs (not necessarily connected) can be constructed out
of a given set V = {v1 , v2 , K vn } of n vertices?
n ( n −1)
n ( n − 1)
(a) (b) 2n (c) n! (d) 2 2
2

2.16 What is the minimum number of stacks of size n required to implement a queue
of size n?
(a) One (b) Two (c) Three (d) Four
2.17 What is printed by the print statements in the program P1 assuming call by
reference parameter passing?
Program P1()
{
x=10;
y=3;
func1(y,x,x);
print x;
print y;
}
func1(x,y,z)
{
y=y+4;
z=x+y+z;
}
(a) 10, 3 (b) 31, 3
(c) 27, 7 (d) None of the above

2.18 Consider the following three C functions:


[P1] int*g(void)
{
intx=10;
return(&x);
}

[P2] int*g(void)
{
int*px;
*px=10;
return px;
}
[P3] int*g(void)
{
int*px
px =(int*)malloc (size of (int));
*px=10;
return px;
}
Which of the above three functions are likely to cause problems with pointers?
(a) Only P3 (b) Only P1 and P3
(c) Only P1 and P2 (d) P1, P2 and P3

2.19 Consider the following program


Program P2
var n:int:
procedure W(var x:int)
begin
x=x+1;
printx;
end

procedure D
begin
var n:int;
n=3;
W(n);
End
begin \\begin P2
n = 10;
D;
end
If the language has dynamic scooping and parameters are passed by reference,
what will be printed by the program?
(a) 10 (b) 11 (c) 3
(d) None of the above

2.20 Which of the following does not interrupt a running process?


(a) A device (b) Timer
(c) Scheduler process (d) Power failure

2.21 Consider a machine with 64 MB physical memory and a 32-bit virtual address
space. If the page size is 4KB, what is the approximate size of the page table?
(a) 16 MB (b) 8 MB (c) 2 MB (d) 24 MB
2.22 Consider Peterson’s algorithm for mutual exclusion between two concurrent
processes i and j. The program executed by process is shown below.
repeat
flag[i]=true;
turn=j;
while (P) do no-op;
Enter critical section, perform actions, then
exit critical section
Flag[i]=false;
Perform other non-critical section actions.
Until false;
For the program to guarantee mutual exclusion, the predicate P in the while loop
should be
(a) flag[j]=true and turn=i (b) flag[j]=true and turn=j
(c) flag[i]=true and turn=j (d) flag[i]=true and turn=i

2.23 R(A,B,C,D) is a relation. Which of the following does not have a lossless join,
dependency preserving BCNF decomposition?
(a) A  B, B  CD (b) A  B, B  C, C  D
(c) AB  C, C  AD (d) A  BCD

2.24 Which of the following relational calculus expressions is not safe?

(a) {t ∃u ∈ R (t  A = u  A ) ∧ ¬∃s ∈ R


1 2 (t  A = s  A )}
(b) {t ∀u ∈ R (u  A = " x " ⇒ ∃s ∈ R (t  A = s  A ∧ s  A = u  A ))}
1 2

(c) {t ¬ (t ∈ R )} 1

(d) {t ∃u ∈ R (t  A = u  A ) ∧ ∃s ∈ R


1 2 (t  A = s  A )}

2.25 Consider a relation geq which represents “greater than or equal to”, that is,
(x,y)∈ geq only if y≥x.
create table geq
( Ib integer not null
ub integer not null
primary key 1b
foreign key (ub) references geq on delete cascade )
Which of the following is possible if a tuple (x,y) is deleted?
(a) A tuple (z,w) with z > y is deleted
(b) A tuple (z,w) with z > x is deleted
(c) A tuple (z,w) with w < x is deleted
(d) The deletion of (x,y) is prohibited

SECTION B

This section consists of TWENTY questions of FIVE marks each. Any FIFTEEN out of
these questions have to be answered on the Answer Book provided.

3. (a) prove that powerset ( A ∩ B ) = powerset(A)∩powerset(B)

(b) Let sum(n)=0+1+2+…..+n for all natural numbers n. give an induction proof
to show that the following equation is true for all natural numbers m and n:
sum(m+n)=sum(m)+sum(n)+mn

4. Consider the function h: N×N  N so that h (a,b) = (2a + 1) 2b − 1, where N


={0,1,2,3,…..} is the set of natural numbers.
(a) Prove that the function h is an injection (one-one).
(b) Prove that it is also a Subjection (onto)

5. Construct DFA’s for the following languages:

{
(a) L = w w ∈ {a, b} *, w has baab as a subsring }
(b) L = {w w ∈ {a, b} *, w has an odd number of a's and an odd nuber of b's}

6. { }
Give a deterministic PDA for the language L = ancb2n n ≥ 1 over the alphabet =

∑ = {a, b, c} . Specify the acceptance state.

7. Let a decision problem X be defined as follows:


X: Given a Turing machine M over Σ and nay word w ∈ Σ,
does M loop forever on w?
You may assume that the halting problem of Turing machine is undecidable but
partially decidable.
(a) Show that X is undecidable.
(b) Show that X is not even partially decidable.
8. Consider a disk with following specifications: 20 surface, 1000 tracks/surface, 16
sectors/track, data density 1 KB/sector, rotation speed 3000 rpm. The operating
system initiates the transfer between the disk and the memory sector-wise. Once
the head has been placed on the right track, the disk reads a sector in a single
scan. It reads bits from the sector while the head is passing over the sector. The
read bits are formed into bytes in a serial-in-parallel-out buffer and each byte is
then transferred to memory. The disk writing is exactly a complementary
process.
For parts (c) and (d) below, assume memory read-write time = 0.1 micro-
second/byte, interrupt driven transfer has an interrupt overhead = 0.4
microseconds, the DMA initialization and termination overhead is negligible
compared to the total sector transfer time. DMA requests are always granted.
(a) What is the total capacity of the disk?
(b) What is the data transfer rate?
(c) What is the percentage of time the CPU is required for this disk I/O for byte-
wise interrupts driven transfer?
(d) What is the maximum percentage of time the CPU is held up for this disk I/O
for cycle-stealing DMA transfer?

9. A CPU has 32-bit memory address and a 256 KB cache memory. The cache is
organized as a 4-way set associative cache with cache block size of 16 bytes.
(a) What is the number of sets in the cache?
(b) What is the size (in bits) of the tag field per cache block?
(c) What is the number and size of comparators required for tag matching?
(d) How many address bits are required to find the byte offset within a cache
block?
(e) What is the total amount of extra memory (in bytes) required for the tag
bits?

10. (a) Is the 3-variable function f = ∑ (0,1, 2, 4) its self-dual? Justify your answer.
(b) Give a minimal product-of-sum form of the b output of the following excess-3
to BCD converter.

e8 b8
e4 Excess 3 b4
e2 To BCD b2
e1 b1

11. A sequential circuit takes an input stream of 0’s and 1’s and produces an output
stream of 0’s and 1’s. Initially it replicates the input on its output until two
consecutive 0’s are encountered on the input. From then onward, it produces an
output stream, which is the bit-wise complement of input stream until it
encounters two consecutive 1’s, whereupon the process repeats. An example of
input and output stream is shown below.
The input stream: 101100 01001011 0 11
The desired output: 101100 10110100 0 11

J-K master-slave flip-flops are to be used to design the circuit.


(a) Give the state transition diagram.
(b) Give the minimized sum-of-product expression for J and K inputs of one of its
state flip-flops.

12. Consider a 5-stage pipeline – IF (Instruction Fetch), ID (Instruction Decode and


register read), EX (Execute), MEM (memory), and WB (Write Back). All (memory
or register) reads take place in the second phase of a clock cycle and writes occur
in the first phase of the clock cycle. Consider the execution of the following
instruction sequence:
11: sub r2, r3, r4; /* r2  r3 – r4 */
12: sub r4, r2, r3; /* r4  r2 – r3 */
13: sw r2, 100(r1) /* M[r1+100] r2 */
14: sub r3, r4, r2; /* r3  r4 – r2 */
(a) Show all data dependencies between the four instructions.
(b) Identify the data hazards.
(c) Can all hazards be avoided by forwarding in this case?

13. Consider the following C program:


void abc(char*s)
{
if(s[0]==’\0’)return;
abc(s+1);
abc(s+1);
printf(“%c”,s[0]);
}
main()
{ abc(“123”)
}
(a) What will be the output of the program?
(b) If abc(s) is called with a null-terminated string s of length n characters (not
counting the null (‘\0’) character), how many characters will be printed by
abc(s)?

14. (a) Insert the following keys one by one into a binary search tree in the order
specified.
15, 32, 20, 9, 3, 25, 12, 1
Show the final binary search tree after the insertions.
(b) Draw the binary search tree after deleting 15 from it.
(c) Complete the statements S1, S2 and S3 in the following function so that the
function computes the depth of a binary rooted at t.
typedef struct tnode{
int key;
struct tnode *left, *right;
} *Tree;
int depth(Tree t)
{
int x,y;
it (t ==NULL) return0;
x=depth(t  left);
S1: ____________;
S2: if(x>y) return _____________:
S3: else return _____________;
}

15. Consider a weighted undirected graph with vertex set V = {n1,n2,n3,n4,n5,n6}


and edge set
E={(n1,n2,2),(n1,n3,8),(n1,n6,3),(n2,n4,4),(n2,n5,12),(n3,n4,7),(n4,n5,9),
(n4,n6,4)}. The third value in each tuple represents the weight of the edge
specified in the tuple.
(a) List the edges of a minimum spanning tree of the graph.
(b) How many distinct minimum spanning trees does this graph have?
(c) Is the minimum among the edge weights of a minimum spanning tree unique
overall possible minimum spanning trees of a graph?
(d) Is the maximum among the edge weights of a minimum spanning tree
unique over all possible minimum spanning trees of a graph?

16. Consider the following grammar with terminal alphabet ∑ {a,(, ), +, *} and start
symbol E. The production rules of the grammar are:
E  aA
E  (E)
A  +E
A  *E
Aε
(a) Compute the FIRST and FOLLOW sets for E and A.
(b) Complete the LL(1) parse table for the grammar.
17. The syntax of the repeat-until statement is given by the following grammar
S  repeat S1 until E
Where E stands for expressions, S and S1 stand for statement. The non-terminals
S and S1 have an attribute code that represents generated code. The non-
terminal E has two attributes. The attribute code represents generated code to
evaluate the expression and store its truth value in a distinct variable, and the
attribute varName contains the name of the variable in which the truth value is
stored? The truth-value stored in the variable is 1 if E is true, 0 if E is false.
Give a syntax-directed definition to generate three-address code for the repeat-
until statement. Assume that you can call a function newlabel( ) that returns a
distinct label for a statement. Use the operator ‘\\’ to concatenate two strings and
the function gen(s) to generate a line containing the string s.

18. (a) Remove left-recursion from the following grammar:


S  Sa| Sb | a | b
(b) Consider the following grammar:
S  aSbS| bSaS |ε
Construct all possible parse trees for the string abab. Is the grammar
ambiguous?

19. Two concurrent processes P1 and P2 want to use two resources R1 and R2 in a
mutually exclusive manner. Initially, R1 and R2 are free. The programs executed
by the two processes are given below.
Program for P1:
S1: While (R1 is busy) do no-op;
S2: Set R1  busy;
S3: While (R2 is busy) do no-op;
S4: Set R2  busy;
S5: Use R1 and R2;
S6: Set R1  free;
S7: Set R2  free;
Program for P2:
Q1: While (R1 is busy) do no-op;
Q2: Set R1  busy;
Q3: While (R1 is busy) do no-op;
Q4: Set R1  busy;
Q5: Use R1 and R2;
Q6: Set R2  free;
Q7: Set R1  free;
(a) Is mutual exclusion guaranteed for R1 and R2? If not, show a possible
interleaving of the statements of P1 and P2 such that mutual exclusion is
violated (i.e., both P1 and P2 use R1 or R2 at the same time).
(b) Can deadlock occur in the above program? If yes, show a possible
interleaving of the statements of P1 and P2 leading to deadlock.
(c) Exchange the statements Q1 and Q3 and statements Q2 and Q4. Is mutual
exclusion guaranteed now? Can deadlock occur?

20. Consider a disk with the 100 tracks numbered from 0 to 99 rotating at 3000 rpm.
The number of sectors per track is 100. the time to move the head between two
successive tracks is 0.2 millisecond.
(a) Consider a set of disk requests to read data from tracks 32, 7, 45, 5 and 10.
Assuming that the elevator algorithm is used to schedule disk requests, and
the head is initially at track 25 moving up (towards larger track numbers),
what is the total seek time for servicing the requests?
(b) Consider an initial set of 100 arbitrary disk requests and assume that no new
disk requests arrive while servicing these requests. If the head is initially at
track 0 and the elevator algorithm is used to schedule disk requests, what is
the worst case time to complete all the requests?

21. Consider the relation examinee (regno, name, score), where regno is the primary
key to score is a real number.
(a) Write a relational algebra using (∏,σ,ρ,×) to find the list of names which
appear more than once in examinee.
(b) Write an SQL query to list the regno of examinees who have a score greater
than the average score.
(c) Suppose the relation appears (regno, centr_code) specifies the center where
an examinee appears. Write an SQL query to list the centr_code having an
examinee of score greater than 80.

22. We wish to construct a B+ tree with fan-out (the number of pointers per node)
equal to 3 for the following set of key values:
80, 50, 10, 70, 30, 100, 90
Assume that the tree is initially empty and the values are added in the order
given.
(a) Show the tree after insertion of 10, after insertion of 30, and after insertion
of 90. Intermediate trees need not be shown.
(b) The key values 30 and 10 are now deleted from the tree in that order. Show
the tree after each deletion.
Hosted at www.educationobserver.com

For more papers visit www.educationobserver.com/forum

SECTION A
1. This question consists of TWENTY-FIVE sub-questions (1.1 – 1.25) of ONE mark
each. For each of these sub-questions, four possible alternatives, A, B, C and D
are provided. Choose the most appropriate alternative and darken its bubble on
the Objective Response Sheet (ORS) against the corresponding sub-question
number using a soft HB pencil. Do not darken more than one bubble for any
sub-question. Do not use the ORS for any rough work. You may use the answer
book (last few pages) for any rough work.

1 1 
1.1 The rank of the matrix   is
0 0
(a) 4 (b) 2 (c) 1 (d) 0

1.2 The trapezoidal rule for integration gives exact result when the integrand is a
polynomial of degree
(a) 0 but not 1 (b) 1 but not 0 (c) 0 or 1 (d) 2

1.3 ( ) ( )
The solution to the recurrence equation T 2k = 3T 2k −1 + 1, T (1) = 1 is

(a) 2 k
(b)
(3 k +1
−1 ) k
(c) 3log2
k
(d) 2log3
2

1.4 The minimum number of colours required to colour the vertices of a cycle with n
nodes in such a way that no two adjacent nodes have the same colour is
n
(a) 2 (b) 3 (c) 4 (d) n − 2   + 2
2 

1.5 In the worst case, the number of comparisons needed to search a singly linked
list of length n for a given element is
n
(a) log n (b) (c) log2n − 1 (d) n
2

1.6 Which of the following is true?


(a) The set of all rational negative numbers forms a group under multiplication.
(b) The set of all non-singular matrices forms a group under multiplication.
(c) The set of all matrices forms a group under multiplication.
(d) Both B and C are true.

1.7 The language accepted by a Pushdown Automaton in which the stack is limited to
10 items is best described as
(a) Context free (b) Regular
(c) Deterministic Context free (d) Recursive
1.8 “If X then Y unless Z” is represented by which of the following formulas in
prepositional logic? (“ ¬ “, is negation, “∧” is conjunction, and “” is implication)
(a) (X∧¬Z)Y (b) (X∧Y)¬Z (c) X(Y∧¬Z) (d) (XY)∧¬Z

1.9 A device employing INTR line for device interrupt puts the CALL instruction on the
data bus while
(a) INTA is active (b) HOLD is active
(c) READY is active (d) None of the above

1.10 In 8085 which of the following modifies the program counter?


(a) Only PCHL instruction (b) Only ADD instructions
(c) Only JMP and CALL instructions (d) All instructions

1.11 In serial data transmission, every byte of data is padded with a ‘0’ in the
beginning and one or two ‘1’s at the end of byte because
(a) Receiver is to be synchronized for byte reception
(b) Receiver recovers lost ‘0’s and ‘1’s from these padded bits
(c) Padded bits are useful in parity computation
(d) None of the above

1.12 Minimum sum of product expression for f(w,x,y,z) shown in Karnaugh-map below
is
wx
00 01 11 10
yz
00 0 1 1 0

01 x 0 0 1

11 x 0 0 1

10 0 1 1 x

(a) xz + y ′z (b) xz ′ + zx ′
(c) x ′y + zx ′ (d) None of the above

1.13 Which of the following is not a form of memory?


(a) instruction cache (b) instruction register
(c) instruction opcode (d) translation look-a-side buffer
1.14 The decimal value 0.25
(a) is equivalent to the binary value 0.1
(b) is equivalent to the binary value 0.01
(c) is equivalent to the binary value 0.00111…
(d) cannot be represented precisely in binary

1.15 The 2’s complement representation of the decimal value –15 is


(a) 1111 (b) 11111 (c) 111111 (d) 10001

1.16 Sign extension is a step in


(a) floating point multiplication
(b) signed 16 bit integer addition
(c) arithmetic left shift
(d) converting a signed integer from one size to another

1.17 In the C language


(a) At most one activation record exists between the current activation record
and the activation record for the main
(b) The number of activation records between the current activation record and
the activation record fro the main depends on the actual function calling
sequence.
(c) The visibility of global variables depends on the actual function calling
sequence.
(d) Recursion requires the activation record for the recursive function to be
saved on a different stack before the recursive fraction can be called.

1.18 The results returned by function under value-result and reference parameter
passing conventions
(a) Do not differ
(b) Differ in the presence of loops
(c) Differ in all cases
(d) May differ in the presence of exception

1.19 Relation R with an associated set of functional dependencies, F, is decomposed


into BCNF. The redundancy (arising out of functional dependencies) in the
resulting set of relations is
(a) Zero
(b) More than zero but less than that of an equivalent 3NF decomposition
(c) Proportional to the size of F+
(d) Indetermine
1.20 With regard to the expressive power of the formal relational query languages,
which of the following statements is true?
(a) Relational algebra is more powerful than relational calculus
(b) Relational algebra has the same power as relational calculus.
(c) Relational algebra has the same power as safe relational calculus.
(d) None of the above

1.21 In 2’s complement addition, overflow


(a) is flagged whenever there is carry from sign bit addition
(b) cannot occur when a positive value is added to a negative value
(c) is flagged when the carries from sign bit and previous bit match
(d) None of the above

1.22 Which of the following scheduling algorithms is non-preemptive?


(a) Round Robin (b) First-In First-Out
(c) Multilevel Queue Scheduling
(d) Multilevel Queue Scheduling with Feedback

1.23 The optimal page replacement algorithm will select the page that
(a) Has not been used for the longest time in the past.
(b) Will not be used for the longest time in the future.
(c) Has been used least number of times.
(d) Has been used most number of times.

1.24 In the absolute addressing mode


(a) the operand is inside the instruction
(b) the address of the operand is inside the instruction
(c) the register containing the address of the operand is specified inside the
instruction
(d) the location of the operand is implicit

1.25 Maximum number of edges in a n-node undirected graph without self loops is
n ( n − 1) ( n + 1) ( n )
(a) n2 (b) (c) n - 1 (d)
2 2
2. This question consists of TWENTY-FIVE sub-questions (2.1 – 2.25) of TWO marks
each. For each of these sub-questions, four possible alternatives, A, B, C and D
are provided. Choose the most appropriate alternative and darken its bubble on
the Objective Response Sheet (ORS) against the corresponding sub-question
number using a soft HB pencil. Do not darken more than one bubble for any
sub-question. Do not use the ORS for any rough work. You may use the answer
book (last few pages) for any rough work.
2.1 Consider the following logic circuit whose inputs are functions f1, f2, f3 and output
is f.

f1(x,y,z)
f2(x,y,z) f(x,y,z)

f3(x,y,z)=?

Given that
f1 ( x, y , z ) =∑ (0,1, 3,5) ,
f ( x, y , z ) = ∑ ( 6, 7 ) , and
2

f ( x, y , z ) = ∑ (1, 4,5) ,

f3 is
(a) ∑ (1, 4,5) (b) ∑ (6, 7)
(c) ∑ (0,1, 3,5) (d) None of the above

2.2 Consider the following multiplexor where 10, 11, 12, 13 are four data input lines
selected by two address line combinations A1A0 =00,01,10,11 respectively and f
is the output of the multiplexor. EN is the Enable input.

10
11 4 TO 1
12 multiplexor
13 OUTPUT f(x,y,z)=?
A1
A0
EN

The function f(x,y,z) implemented by the above circuit is


(a) xyz ′ (b) xy+z
(c) x+y (d) None of the above

2.3 (
Let f(A,B) = A′ + B. Simplified expression for function f f ( x + y , y ) , z is )
(a) x ′ + z
(b) xyz
(c) xy ′ + z
(d) None of the above
2.4 What are the states of the Auxillary Carry (AC) and Carry Flag (CY) after
executing the following 8085 program?
MVI H, 5DH
MIV L, 6BH
MOV A, H
ADD L
(a) AC = 0 and CY =0 (b) AC = 1 and CY =1
(c) AC = 1 and CY =0 (d) AC = 0 and CY =1

2.5 The finite state machine described by the following state diagram with A as
x
starting state, where an arc label is and x stands for 1-bit input and y stands
y
for 2-bit output
0/01

A B C
1/01 1/10
1/10
0/00
0/01

(a) Outputs the sum of the present and the previous bits of the input.
(b) Outputs 01 whenever the input sequence contains 11
(c) Outputs 00 whenever the input sequence contains 10
(d) None of the above

2.6 The performance of a pipelined processor suffers if


(a) the pipeline stages have different delays
(b) consecutive instructions are dependent on each other
(c) the pipeline stages share hardware resources
(d) All of the above

2.7 Horizontal microprogramming


(a) does not require use of signal decoders
(b) results in larger sized microinstructions than vertical microprogramming
(c) uses one bit for each control signal
(d) all of the above

2.8 Consider the following declaration of a two-dimensional array in C:


char a[100][100];
Assuming that the main memory is byte-addressable and that the array is stored
starting from memory address 0, the address of a [40][50] is
(a) 4040 (b) 4050 (c) 5040 (d) 5050
2.9 The number of leaf nodes in a rooted tree of n nodes, with each node having 0 or
3 children is:

(a)
n
(b)
( n − 1) (c)
( n − 1) (d)
(2n + 1)
2 3 2 3

2.10 Consider the following algorithm for searching for a given number x in an
unsorted array A[l..n] having n distinct values:
1. Choose an i uniformly at random from l..nl
2. If A[i]=x then Stop else Goto 1;
Assuming that x is present A, what is the expected number of comparisons made
by the algorithm before it terminates?
n
(a) n (b) n - 1 (c) 2n (d)
2

2.11 The running time of the following algorithm


Procedure A(n)
If n<=2 return(1) else return (A( n ));
Is best described by
(a) O(n) (b) O(log n) (c) O(log log n) (d) O(1)

2.12 A weight-balanced tree is a binary tree in which for each node, the number of
nodes in the let sub tree is at least half and at most twice the number of nodes in
the right sub tree. The maximum possible height (number of nodes on the path
from the root to the furthest leaf) of such a tree on n nodes is best described by
which of the following?
(a) log2n (b) log 4 n (c) log3n (d) log3 n
3 2

2.13 The smallest finite automaton which accepts the language


{x length of x is divisible by 3} has
(a) 2 states (b) 3 states (c) 4 states (d) 5 states

2.14 Which of the following is true?


(a) The complement of a recursive language is recursive.
(b) The complement of a recursively enumerable language is recursively
enumerable.
(c) The complement of a recursive language is either recursive or recursively
enumerable.
(d) The complement of a context-free language is context-free
sion of this test paper at http://forum.gatementor.

X  3
2.15 The Newton-Raphson iteration X n +1 =  n + can be used to solve the
 2  (2 X n )
equation
(a) X 2 = 3 (b) X 3 = 3 (c) X 2 = 2 (d) X 3 = 2

2.16 Four fair coins are tossed simultaneously. The probability that at least one head
and one tail turn up is
1 1 7 15
(a) (b) (c) (d)
16 8 8 16

2.17 The binary relation S = φ (empty set) on set A = {1,2,3} is


(a) Neither reflexive nor symmetric (b) Symmetric and reflexive
(c) Transitive and reflexive (d) Transitive and symmetric

2.18 The C language is:


(a) A context free language (b) A context sensitive language
(c) A regular language
(d) Parsable fully only by a Turing machine

2.19 To evaluate an expression without any embedded function calls


(a) One stack is enough
(b) Two stacks are needed
(c) As many stacks as the height of the expression tree are needed
(d) A Turning machine is needed in the general case

2.20 Dynamic linking can cause security concerns because


(a) Security is dynamic
(b) The path for searching dynamic libraries is not known till runtime
(c) Linking is insecure
(d) Cryptographic procedures are not available for dynamic linking

2.21 Which combination of the following features will suffice to characterize an OS as a


multi-programmed OS? (A) More than one program may be loaded into main
memory at the same time for execution. (B) If a program waits for certain events
such as I/O, another program is immediately scheduled for execution. (C) If the
execution of a program terminates, another program is immediately scheduled
for execution.
(a) A (b) A and B (c) A and C (d) A, B and C
2.22 In the index allocation scheme of blocks to a file, the maximum possible size of
the file depends on
(a) the size of the blocks, and the size of the address of the blocks.
(b) the number of blocks used for the index, and the size of the blocks.
(c) the size of the blocks, the number of blocks used for the index, and the size
of the address of the blocks.
(d) None of the above

2.23 A B+ - tree index is to be built on the Name attribute of the relation STUDENT.
Assume that all student names are of length 8 bytes, disk blocks are of size 512
bytes, and index pointers are of size 4 bytes. Given this scenario, what would be
the best choice of the degree (i.e. the number of pointers per node) of the B+ -
tree?
(a) 16 (b) 42 (c) 43 (d) 44

2.24 Relation R is decomposed using a set of functional dependencies, F, and relation


S is decomposed using another set of functional dependencies, G. One
decomposition is definitely BCNF, the other is definitely. 3NF, but it is not known
which is which. To make a guaranteed identification, which one of the following
tests should be used on the decompositions? (Assume that the closures of F and
G are available).
(a) Dependency-preservation (b) Lossless-join
(c) BCNF definition (d) 3NF definition

2.25 From the following instance of a relation schema R(A,B,C), we can conclude that:

A B C
1 1 1
1 1 0
2 3 2
2 3 2

(a) A functionally determines B and B functionally determines C


(b) A functionally determines B and B does not functionally determines C
(c) B does not functionally determines C
(d) A does not functionally determines B and B does not functionally determines
C
SECTION B
This section consists of TWENTY questions of FIVE marks each. Any FIFTEEN out of
these questions have to be answered on the Answer Book provided.

3. Let A be a set of n(>0) elements. Let Nr be the number of binary relations on A


and let Nf be the number of functions from A to A.
(a) Give the expression for Nr in terms of n.
(b) Give the expression for Nf in terms of n.
(c) Which is larger for all possible n, Nr or Nf?

4. (a) S = { 1, 2 }
, 2,1 is binary relation on set A = {1,2,3}. Is it irreflexive? Add
the minimum number of ordered pairs to S to make it an equivalence
relation. Give the modified S.
(b) Let S = {a,b} and let (S) be the powerset of S. Consider the binary
relation ‘⊆ (set inclusion)’ on (S). Draw the Hasse diagram corresponding
to the lattice ((S),⊆)

5. (a) Obtain the eigen values of the matrix

1 2 34 49 
 
0 2 43 94 
A=
0 0 −2 104
 
0 0 0 −1 

(b) Determine whether each of the following is a tautology, a contradiction, or


neither (“∨” is disjunction, “∧” is conjunction, “→” is implication, “¬” in
negation, and “↔” is biconditional (if and only if).
(i) A ↔ ( A ∨ A)

(ii) ( A ∨ B) → B
(iii) A ∧ (¬ ( A ∨ B )

6. Draw all binary trees having exactly three nodes labeled A, B and C on which
Preorder traversal gives the sequence C,B,A.

7. (a) Express the function f ( x, y , z ) = xy ′ + yz ′ with only one complement operation


and one or more AND/OR operations. Draw the logic circuit implementing the
expression obtained, using a single NOT gate and one or more AND/OR
gates.
(b) Transform the following logic circuit (without expressing its switching
function) into an equivalent logic circuit that employs only 6 NAND gates
each with 2-inputs.

8. Consider the following circuit. A = a2a1a0 and B = b2b1b0 are three bit binary
numbers input to the circuit. The output is Z = z3z2z1z0. R0, R1 and R2 are
registers with loading clock shown. The registers are loaded with their input data
with the falling edge of a clock pulse (signal CLOCK shown) and appears as
shown. The bits of input number A, B and the full adders are as shown in the
circuit. Assume Clock period is greater than the settling time of all circuits.
A B
CLOCK

REG R0
(6 – bit) b2 a2 b1 a1 b0 a0

EA 0

REG R1
(6 – bit) b2 a2 b1 a1

EA

REG R2 b2 a2
(5 – bit)

EA

z1 z0
z2
Z=z3

(a) For 8 clocks pulses on the CLOCK terminal and the inputs A, B as shown,
obtain the output Z (sequence of 4-bit values of Z). Assume initial contents
of R0, R1 and R2 as all zeros.

A= 110 011 111 101 000 000 000 000


B= 101 101 011 110 000 000 000 000
Clock No 1 2 3 4 5 6 7 8
(b) What does the circuit implement?
9. Consider the following 32-bit floating-point representation scheme as shown in
the formal below. A value is specified by 3 fields, a one bit sign field (with 0 for
positive and 1 for negative values), a 24 bit fraction field (with the binary point
being at the left end of the fraction bits), and a 7 bit exponent field (in excess-64
signed integer representation, with 16 being the base of exponentiation). The
sign bit is the most significant bit.

1 24 ?

sign fraction exponent

(a) It is required to represent the decimal value –7.5 as a normalized floating


point number in the given format. Derive the values of the various fields.
Express your final answer in the hexadecimal.
(b) What is the largest values that can be represented using this format?
Express your answer as the nearest power of 10.

10. In a C program, an array is declared as float A[2048]. Each array element is 4


Bytes in size, and the starting address of the array is 0×00000000. This program
is run on a computer that has a direct mapped data cache of size 8 Kbytes, with
block (line) size of 16 Bytes.
(a) Which elements of the array conflict with element A[0] in the data cache?
Justify your answer briefly.
(b) If the program accesses the elements of this array one by one in reverse
order i.e., starting with the last element and ending with the first element,
how many data cache misses would occur? Justify your answer briefly.
Assume that the data cache is initially empty and that no other data or
instruction accesses are to be considered.

11. The following recursive function in C is a solution to the Towers of Hanoi problem.
Void move (int n, char A, char B, char C)
{
if (………………………………….) {
move (………………………………….);
printf(“Move disk %d from pole %c to pole %c\n”, n, A,C);
move (………………………………….);
Fill in the dotted parts of the solution.
12. Fill in the blanks in the following template of an algorithm to compute all pairs
shortest path lengths in a directed graph G with n*n adjacency matrix A.
A[i,j]equals if there is an edge in G from i to j, and 0 otherwise. Your aim in filling
in the blanks is to ensure that the algorithm is correct.
INITIALIZATION: For i = 1 … n
{For j = 1 … n
{ if A[i,j]=0 then P[i,j] = _______ else P[i,j] =____;}
ALGORITHM: For i = 1 …n
{ For j = 1 …n
{For k = 1 …n
{P[__,___]=min{_______,_______};}
}
}
(a) Copy the complete line containing the blanks in the Initialization step and fill
in the blanks.
(b) Copy the complete line containing the blanks in the Algorithm step and fill in
the blanks.
(c) Fill in the blank: The running time of the Algorithm is O(____).

13. (a) In how many ways can a given positive integer n ≥ 2 be expressed as the
sum of 2 positive integers (which are not necessarily distinct). For example,
for n = 3, the number of ways is 2, i.e., 1+2, 2+1. Give only the answer
without any explanation.
(b) In how many ways can a given positive integer n ≥ 3 be expressed as the
sum of 3 positive integers (which are not necessarily distinct). For example,
for n = 4, the number of ways is 3, i.e., 1+2+1, 2+1+1. Give only the
answer without any explanation.
(c) In how many ways can a given positive integer n ≥ k be expressed as the
sum of k positive integers (which are not necessarily distinct)? Give only the
answer without explanation.

14. The aim of the following question is to prove that the language {M | M is the
code of a Turing Machine which, irrespective of the input, halts and outputs a 1},
is undecidable. This is to be done by reducing form the language
{ }
M ′, x M ′ halts on x , which is known to be undecidable. In parts (a) and (b)
describe the 2 main steps in the construction of M. in part (c) describe the key
propery which relates the behaviour of M on its input w to the behaviour of M ′ on
x.
(a) On input w, what is the first step that M must make?
(b) On input w, based on the outcome of the first step, what is the second step
that M must make?
(c) What key property relates the behaviour of M on w to the behaviour of M ′ on
x?
15. A university placement center maintains a relational database of companies that
interview students on campus and make job offers to those successful in the
interview. The schema of the database is given below:
COMPANY (cname, clocation) STUDENT (scrollno, sname, sdegree)
INTERVIEW (cname, srollno, idate) OFFER (cname,srollno, osalary)
The COMPANY relation gives the name and location of the company. The
STUDENT relation gives the student’s roll number, name and the degree program
for which the student is registered in the university. The INTERVIEW relation
gives the date on which a students is interviewed by a company. The OFFER
relation gives the salary offered to a student who is successful in a company’s
interview. The key for each relation is indicated by the underlined attributes.
(a) Write relational algebra expressions (using only the operator ,σ,π,∪,− )
for the following queries:
(i) List the rollnumbers and names of those students who attended at least one
interview but did not receive any job offer.
(ii) List the rollnumbers and names of students who went for interviews and
received job offers from every company with which they interviewed.
(b) Write an SQL query to list, for each degree program in which more than five
students were offered jobs, the name of the degree and the average offered
salary of students in this degree program.

16. For relation R = (L, M, N , O, P), the following dependencies hold:


M  O NO  P P  L and L  MN
R is decomposed into R1 =(L, M, N , P) and R2 = (M, O).
(a) Is the above decomposition a lossless-join decomposition? Explain.
(b) Is the above decomposition dependency-preserving? If not, list all the
dependencies that are not preserved.
(c) What is the highest normal form satisfied by the above decomposition?

17. (a) The following table refers to search times for a key in B-trees and B+-trees.

B-tree B+-tree
Successful Search Unsuccessful search Successful Search Unsuccessful search
X1 X2 X3 X4

A successful search means that the key exists in the database and
unsuccessful means that it is not present in the database. Each of the entries
X1, X2, X3 and X4 can have a value of either Constant or Variable. Constant
means that the search time is the same, independent of the specific key
value, where Variable means that it is dependent on the specific key value
chosen for the search.
Give the correct values for the entries X1, X2, X3 and X4 (for example X1 =
Constant, X2= Constant, X3 = Constant, X4= Constant).
(b) Relation R(A,B) has the following view defined on it:
CREATE VIEW V AS
(SELECT R1.A,R2.B
FROM R AS R1, R AS R2
WHERE R1.B=R2.A)

(i) The current contents of relation R are shown below. What are the contents of
the view V?
A B
1 2
2 3
2 4
4 5
6 7
6 8
9 10

(ii) The tuples (2,11) and (11,6) are now inserted into R. What are the additional
tupels that are inserted in V?

18. (a) Draw the process state transition diagram of an OS in which (i) each process
is in one of the five states: created, ready, running, blocked (i.e. sleep or
wait), or terminated, and (ii) only non-preemptive scheduling is used by the
OS. Label the transitions appropriately.
(b) The functionality of atomic TEST-AND-SET assembly language instruction is
given by the following C function.
int TEST-AND-SET (int *x)
{
int y;
A1:y=*x;
A2:*x=1;
A3:return y;
}
(i) Complete the following C functions for implementing code for entering and
leaving critical sections based on the above TEST-AND-SET instruction.
int mutex=0;
void enter-cs()
{
while (…………………………………);
}
void leave-cs()
{
…………………………………..;
}
(ii) Is the above solution to the critical section problem deadlock free and
starvation-free?
(iii) For the above solution, show by an example that mutual exclusion is not
ensured if TEST-AND-SET instruction is not atomic.

19. A computer system uses 32-bit virtual address, and 32-bit physical address. The
physical memory is byte addressable, and the page size is 4 kbytes. It is decided
to use two level page tables to translate from virtual address to physical address.
Equal number of bits should be used for indexing first level and second level page
table, and the size of each page table entry is 4 bytes.
(a) Give a diagram showing how a virtual address would be translated to a
physical address.
(b) What is the number of page table entries that can be contained in each
page?
(c) How many bits are available for storing protection and other information in
each page table entry?

20. The following solution to the single producer single consumer problem uses
semaphores for synchronization.
#define BUFFSIZE 100
buffer buf[BUFFSIZE];
int first=last=0;
semaphore b_full=0;
semaphore b_empty=BUFFSIZE;

void producer()
{
while (1) {
produce an item;
p1: …………………..;
put the item into buff (first);
first=(first+1)%BUFFSIZE;
p2: …………………..;
}
}
void consumer()
{
Hosted at www.educationobserver.com

For more papers visit www.educationobserver.com/forum

while (1) {
c1:……………………..
take the item from buf[last];
last=(last+1)%BUFFSIZE;
c2: ……………………..;
consume the item;
}
}

(a) Complete the dotted part of the above solution.


(b) Using another semaphore variable, insert one line statement each
immediately after p1, immediately before p2, immediately after c1, and
immediately before c2 so that the program works correctly for multiple
procedures and consumers.

21. We require a four state automaton to recognize the regular expression


( a b ) * abb.
(a) Give an NFA for this purpose.
(b) Give a DFA for this purpose.

22. (a) Construct all the parse trees corresponding to i + j * k for the grammar
E  E+E
E  E*E
E  id

(b) In this grammar, what is the precedence of the two operators * and +?
(c) If only one parse tree is desired for any string in the same language, what
changes are to be made so that the resulting LALR(1) grammar is non-
ambiguous?

You might also like